You are on page 1of 70

MCQs for political science optional aspirants

Q. 1. The correct sequence in Game theory is:


(a) gamechoiceplayerequilibrium
(b) payoffs strategies system game
(c) payoffs choice player game
(d) systems equilibrium game player
Ans. (a)
Q. 2. Which one of the following statements is not correct?
(a) Diplomacy is an instrument of foreign policy
(b) Major international agreements are usually negotiated by foreign ministers
(c) Diplomats do not play now as great a role in international negotiations as they once did
(d) Diplomats now get greater latitude than they enjoyed 19th Century
Ans. (d)
Q. 3. Which one does not contribute to the formation of International Government?
(a) Expansion of international trade
(b) Establishment of international organization
(c) Inter-state rivalry
(d) Means of communication
Ans. (c)
Q. 4. In his realistic theory of International politics Morgenthaus autonomy of political sphere denotes
(a) autonomy of the State in international sphere
(b) autonomy of the decision-makers vis-a vis-the legislature
(c) autonomy of the political action in complete disregard to other standards of thought
(d) autonomy of the policy-makers vis-a vis other
agencies or organs of the government
Ans. (c)
Q. 5. The Marxists perceive international politics as:
(a) a struggle for power between the bourgeois and socialist systems with the hope of inevitable triumph of international proletarianism
(b) a struggle for power between two nations or group of nations that can be managed by maintaining balances of power
(c) Interactions among States that can be peacefully
(d) Interactions among States guided and molded by leaders of those States
Ans. (a)
Q. 6. Decision making theorists focus on:
(a) foreign policy decisions made by a State
(b) impact of domestic politics on foreign policy decisions
(c) Specific decisions makers who speak and act in the name of a State
(d) Actual or objective environment that influence foreign policy decisions
Ans. (d)
Q. 7. Who among the following is the pioneer of the theory of neo-realism?
(a) Kenneth Waltz
(b) Hans Morgenthau
(c) Barry Buzari
(d) Raul Prebish
Ans. (a)
Q. 8. In the positive sum game:
(a) what one actor wins, the other loses
(b) gains and losses are not necesbji1y equal and both the sides may gain
(c) gains and losses are necessarily equal and both the sides may gain
(d) both the actors lose
Ans (b)
Q. 9. Schellings concepts of mixed motive game richly embodies:
(a) political and economic points of view
(b) cultural and ideological points of view
(c) psychological and sociological points of view
(d) strategic and military points of view
Ans. (c)
Q. 10. Central feature of Immanuel Wallensteins work is:
(a) world system
(b) nation
(c) regional systems
(d) underdevelopment
Ans. (a)
Q. 11. Which among the following scholars propounded the systems approach towards the study of international relations?
(a) David Easton
(b) Gabriel Almond
(c) C. McClelland

Muftbooks.com

MCQs for political science optional aspirants


(d) J.W. Burton
Ans. (c)
Q. 12. Match List -I with List -II and select the correct answer using the code given below.
List-I List- II
(Scholar) (Theory/Approach)
A. Morgenthau 1. Systems Theory
B. Bernard Cohen 2. Game Theory
C. Merton Kaplan 3. Decision making approach
D. Emile Berd 4. Realist School
Codes:
ABCD
(a) 4 3 1 2
(b) 2 1 3 4
(c) 4 1 3 2
(d) 2 3 1 4
Ans. (a)
Q. 13. Which one of the following approaches or modest of international politics does not give importance to the role of state as an
actor?
(a) Dependency model
(b) Realist Model
(c) Neo-realist model
(d) Interdependence pluralist model
Ans. (a)
Q. 14. Which country was appointed the first Chairman of the Decolonization Committee of the United Nations?
(a) USA
(c) India
(b) Sweden
(d) Japan
Ans. (c)
Q. 15. Consider the following statements in respect of the general systems theory:
1. The general systems theory is highly attractive from the standpoint an empirical research.
2. The theory has been criticized for falling to adequately cater for concept such as political power and influence.
Which of the statement given above is/are correct?
(a) 1 only
(b) 2 only
(c) Both 1 and 2
(d) Neither 1 nor 2
Ans. (c)
Q. 16. Given below are two statements, one labelled as Assertion (A) and the other 1abelledas Reason (R):
Assertion (A): Marxist theory of international is less of a theory and more of a perspective towards understanding of international
relations.
Reason (R): Though Karl Marx has written extensively and he has been identified as a great theorist and philosopher of the modern
times, yet he did not put forward a theory of international relations like Morgenthau, Kaplan and Snyder. In the context of the above
two statements, which one of the following is correct?
Codes:
(a) Both A and R are individually true and R is the correct explanation of A
(b) Both A and R are individually true but R is not the correct explanation of A
(c) A is true but R is false
(d) A is false but R is true
Ans. (b)
Q. 17. Given below are two statements, one labelled as Assertion (A) and the other labelled as Reason (R):
Assertion (A): The decision making theory was developed especially in the realm of foreign policy making.
Reason (R): It concentrates/focuses on the person shaping international events rather than on the international situation:
In the context of the above two statements, which one of the following is correct?
Codes:
(a) Both A and R are individually true and R is the correct explanation of A
(b) Both A and R are individually true but R is not the correct explanation of A
(c) A is true but R is false
(d) A is false but R is true
Ans. (a)
Q. 18. What is the model that deals with a conflict situation where all the rivals are to benefit in different degrees, described as?
(a) Malign conflict situation
(b) Benign conflict situation.
(c) Zero sum situation
(d) Non-zero sum situation
Ans. (c)
Q. 19. Which one of the following is an apt description of Bodins theory of sovereignty?
(a) Absolute sovereignty

Muftbooks.com

MCQs for political science optional aspirants


(b) Limited sovereignty
(c) Political sovereignty
(d) Popular sovereignty
Ans. (a)
Q. 20. Who among the following thinkers criticized the Austinian theory of sovereignty on the ground that law is not the command of
the sovereign but an expression of the communitys sense of right?
(a) Laski
(b) Krabbe
(c) Duguit
(d) Gierke
Ans. (b)
Q. 21. Associations are not persona ficta but real persons existing prior to the State. This view was propounded by:
(a) Hobbes
(b) Hegel
(c) Duguit
(d) Gierke
Ans. (d)
Q. 22. About which of the following thinkers it has been said that his theory of the State is an incongruous mixture of natural rights
and physiological metaphor?
(a) Bradley
(b) Spencer
(c) Bluntschli
(d) Burke
Ans. (b)
Q. 23. Match List- I with List -II and select the correct
List -I List -II
A. The State is the soul 1. Hegel writ large
B. The State is coordinating 2. Weber and adjusting
C. The State has the monopoly of 3. Ernest Barker
legitimate physical force
D. The State is the self realizing 4. Plato and self-actualizing individual
Codes:
ABCD
(a) 4 3 2 1
(b) 5 2 1 4
(c) 1 3 2 1
(d) 4 2 3 1
Ans. (a)
Q. 24. The idea of social contract has been recently revived under a new form by:
(a) Nozick
(b) Rawls
(c) Oakshott
(d) Hannah Arendt
Ans. (b)
Q. 25. The theory of social contract primarily seeks:
(a) to explore the historical origin of the State
(b) to explain the basis of political obligation
(c) to justify the status quo
(d) to bring out a radical transformation of society by revolution
Ans. (b)
Q. 26. Which one of the following statements about Rousseau is correct?
(a) He perfected the theory of Social Contract
(b) He repudiated the theory of Contract altogether
(c) He transformed the theory of Contract into an idealist mode of political discourse
(d) He added nothing to the theories of Hobbes and Locke beyond combining their views
Ans. (c)
Q. 27. The Historical Theory of the Origin of the State was propounded by:
(a) Sir Henry Maine
(b) Triestske
(c) Oppenheimer
(d) Durkheim
Ans. (a)
Q. 28. Which one of the following theories held that the State is a product and manifestation of the irreconcilability of class
antagonisms?
(a) Evolutionary Theory
(b) Anarchist Theory
(c) Marxist-Leninist Theory

Muftbooks.com

MCQs for political science optional aspirants


(d) Guild-Socialist Theory
Ans. (c)
Q. 29. What is the correct order of primacy of the following layers of social structure according to the Materialistic Interpretation
History?
1. Relations of production
2. Ideology
3. Forces of production
4. Legal and Political organizations
Select the correct answer using the codes given below:
(a) 1, 3, 2, 4
(b) 3, 1, 4, 2
(c) 3, 4, 1, 2
(d) 2, 3, 4, 1
Ans. (b)
Q. 30. Which one of the following political thinkers was the first exponent of the liberal theory of States?
(a) John Locke
(b) T.H. Green
(c) Jean Jacques Rousseau
(d) Hobbes
Ans. (a)
Q. 31. By liberty I mean the eager maintenance of that atmosphere in which men have the opportunity to be their best selves. Laski.
Which one Of the following expresses the view implied in the above statement?
(a) Liberty is the absence of restraint
(b) Liberty is what the law permits
(c) Liberty is the ability to do what one desires
(d) Liberty is the maintenance of conditions for the growth of human personality
Ans. (d)
Q. 32. Which one of the following statements about the welfare state is correct?
(a) A welfare state is the same as a socialist state
(b) The welfare state is based on the principles of classical liberalism
(c) The welfare state is collectivist state
(d) The welfare state embodies the principles of neoliberalism
Ans. (b)
Q. 33. Who among the following thinkers held that Human consciousness postulates liberty, liberty involves rights and rights demand
the State?
(a) Hegel
(b) Green
(c) Bosanquet
(d) Laski
Ans. (b)
Q. 34. Rights properly so-called are creations of law properly so called.
This definition of rights is associated with:
(a) Hobbes
(b) Hegel
(c) Bentham
(d) Laski
Ans. (c)
Q. 35. The essence of Natural Rights is that:
(a) they are derived from a historical state to nature
(b) they existed in the state of nature and are not existent in civil society
(c) they are the products of nature conceived as concates natus of physical forces
(d) they are in conformity with universal principles of justice and morality and morality
Ans. (a)
Q. 36. Who among the following used the expression forced to be free in connection with the notion of liberty of the individual?
(a) Rousseau
(b) Locke
(c) Green
(d) Hobhouse
Ans. (a)
Q. 37. The view that property is a natural right of man was propounded by:
(a) Aristotle
(b) St. Augustine
(c) Locke
(d) Hegel
Ans. (c)

Muftbooks.com

MCQs for political science optional aspirants


Q. 38. The positive theory of liberty was advocated by:
(a) John Stuart Mill
(b) T.H. Green
(c) Isaih Berlin
(d) Karl Popper
Ans. (b)
Q. 39. Bentham was a supporter of the theory of:
(a) natural rights
(b) historical rights
(c) legal rights
(d) ethical rights
Ans. (c)
Q. 40. The doctrine of proportionate equality was propounded by:
(a) Aristotle
(b) Rousseau
(c) Marx
(d) Rawls
Ans. (a)
Q. 41. Who among the following thinkers held the view that equality and liberty
are antithetical?
(a) Lord Action
(b) Karl Marx
(c) Harold Laski
(d) G.D.H. Cole
Ans. (a)
Q. 42. Rawls principle of distributive justice is based on the notion of:
(a) desert
(b) difference principle
(c) equality
(d) entitlement
Ans. (b)
Q. 43. The principle of distributive justice was first propounded by:
(a) Aristotle
(b) Godwin
(c) Herbert Spencer
(d) Rawls
Ans. (a)
Q. 44. Match List- I with List- II and select the correct answer by using the codes given below the lists:
List- I List- II
A. Justice is the interest of the stronger 1. Duguit
B. Justice is doing ones own work
which is in consonance with his own nature 2. Thrasymachus
C. Justice is ultimately the principle of social 3. Rawls
solidarity
D. Justice is fairness 4. Plato
5. Spencer
Codes:
ABCD
(a) 2 4 1 3
(b) 3 5 1 2
(c) 2 1 3 4
(d) 5 4 2 1
Ans. (a)
Q. 45. The thesis which seeks to establish the relationship between electoral process and the party system is known as:.
(a) Moscas thesis
(b) Robert Michels theory
(c) Webers doctrine
(d) President
Ans (d)
Q. 46. Which one among the following is regarded as a civil right?
(a) Right to public employment
(b) Freedom of speech and expression
(c) Right to elect and to be elected
(d) Right to property
Ans. (b)
Q. 47. The liberal theory was juristic rather than political in:
(a) Germany

Muftbooks.com

MCQs for political science optional aspirants


(b) England
(c) France
(d) The U.S.A
Ans. (c)
Q. 48. Who among the following has expressed the view that if justice is taken away; the State becomes a band of robbers?
(a) Pluto
(b) Aristotle
(c) St. Augustine
(d) Locke
Ans. (c)
Q. 49. For the removal of the abuses of the factory system Robert Owen advocated:
(a) the organization of a strong trade union movement
(b) a revolutionary change of class relations in society
(c) the organization of producers co-operatives
(d) a reward system based on the principle of to each according to his needs
Ans. (c)
Q. 50. Which of the following statements are consistent with the Pluralist theory of sovereignty?
1. Although the State is also an association, the possession of coercive power by the State endows it with a superior right over other
associations
2. The associations in the State are as real and as self-sufficient as the State itself
3. Law is prior to the State
4. All other associations having valuable social functions to perform are as much sovereign as the State is for its purpose
(a) 1, 2 and 3
(b) 1, 2 and 4
(c) 1, 3 and 4
(d) 2, 3 and 4
Ans. (a)
........................................------------------------------------..............................................
Political Science Quiz
Quiz about Political Science.
practice of political science questions
quiz questions and answers about Political Science
Q. 1. Match List- I with List -II and select the correct answer:
List-I List-II
(Type of Constitution) (Name of the country)
A. Parliamentary, Federal 1. United Kingdom republican
B. Presidential, Federal 2. France republican
C. Parliamentary, Unitary, Monarchial 3. Untied States of America
D. Parliamentary cum presidential unitary 4. India Unitary, republican
5. Nigeria
Codes:
ABCD
(a) 4 3 1 2
(b) 4 3 2 1
(c) 3 4 1 5
(d) 3 5 4 1
Ans. (a)
Q. 2. Parliamentary supremacy is a definite feature of the political system in:
(a) U.K.
(b) India
(c) Canada
(d) Australia
Ans. (a)
Q. 3. Indian Federation closely resembles:
(a) U.S.A
(b) Canada
(c) Australia
(d) Nigeria
Ans. (b)
Q. 4. Which of the following are the characteristics of Parliamentary opposition in Britain according to S.E. Finer?
1. It is unrecognized
2. It is permanent
3. It is representative
4. It is a participant
Select the correct answer using the codes given below:
(a) l and 4
(b) l and 2
(c) 1, 2 and 3

Muftbooks.com

MCQs for political science optional aspirants


(d) 2, 3 and 4
Ans. (d)
Q. 5. Which one of the following statements is not correct?
In the American federal system:
(a) The powers of states are undefined but they are not unlimited
(b) All states possess equal rights and equal representation in the Senate
(c) The federal government can change the name and territory of any constituent state
(d) The residuary powers are vested in the states
Ans. (c)
Q. 6. Which one of the following is not a correct statements?
(a) The Governor General in can refuse his assent to a provincial legislation if the Governor reserves it for his assent
(b) The Indian President can refuse his assent to a state legislation, if the Governor reserves it for his assent
(c) In Australia, the Governor General can refuse State Legislation, if the Governor reserves it for his assent
(d) In the U.S.A., the bills passed by State legislature cannot be reserved for Presidents assent
Ans. (c)
Q. 7. The basic feature of the federal polity of the former Soviet Union was that it was a:
(a) Federal union of multinational states enjoying maximum autonomy
(b) Political contrivance intended to reconcile national unity with maintenance of State rights
(c) Voluntary union of equal Soviet Socialist Republics founded on the principles of nationality
(d) Solid unification of different nationalities into a single centralized State
Ans. (c)
Q. 8. Which one of the following federations did/does not empower its highest curt to have judicial review of the constitutionality of
federal legislation?
(a) India
(b) Former U.S.S.R
(d) Canada
(c) U.S.A
Ans. (b)
Q. 9. The Canadian Federal model provides:
(a) two lists of Legislative powers, one for the Centre and the other for the Province and the residuary powers are vested in the Centre
(b) one list of Legislative powers for the provinces and the residuary powers vested in the Centre
(c) two lists of Legislative powers, one for the Centre, the other for the Provinces and the residuary powers are vested in the Provinces
(d) two lists of equal number of legislative powers, one for Centre and the other for Provinces
Ans. (a)
Q. 10. Which one of the following countries has/had single citizenship?
(a) U.S.A.
(b) Australia
(c) Former U.S.S.R
(d) None of the above
Ans. (d)
Q. 11. Match List -I with List- II and select the correct. answer:
List-I List -II
A. U.S.A. 1. Two states of Courts viz provincial and federal
B. Canada 2. Union of States
C. Nigeria 3. Separate Constitution for States
D. India 4. Separate Electoral Commission for each State
Codes:
ABCD
(a) 1 3 2 4
(b) 3 1 2 4
(c) 3 1 4 2
(d) 1 3 4 2
Ans (c)
Q. 12. Which one of the following statements correctly explains the meaning of Checks and Balance in. the presidential system of
U.S.A?
(a) Checking the growing powers of the federal government and balancing it against the powers of stats
(b) Checking the growing authority of the state, and balancing if against the rights and liberties of the people
(c) The functions of the legislature, executive and judiciary acting as checks on one another, creating a constitutional balance for the
smooth functioning of the government
(d) The power of the Supreme Court in respect of judicial review being kept in check in such a manner as to balance the federal judicial
powers against the judicial powers of the states
Ans. (c)
Q. 13. In which one of the following cases did Marshal, the Chief Justice of the U.S. Supreme Court recognize the powers of Judicial
Review?
(a) McCulloch V. Maryland
(b) Marbury V. Madison
(c) Gibbon V. Ogdeu

Muftbooks.com

MCQs for political science optional aspirants


(d) Scott V. Stanford
Ans (b)
Q. 14. Who is the constitution head of Australia?
(a) The Queen of England,
(b) The Prime Minister of Australia
(c) The Speaker of the Australian Parliament
(d) The Governor General of Australia
Ans. (a)
Q. 15. Which one of the following groups of countries ha been suspended from the membership of the Commonwealth in 1998 and
1999?
(a) Cameroon and Pakistan
(b) Malawi and Sierra Leone
(c) Zambia and Pakistan
(d) Nigeria and Pakistan
Ans. (d)
Q. 16. Which one of the following statements is not correct?
(a) In 1996, the nationalist and communist opposition in the Russian Parliament carried a motion voiding the decision of December
1991 to dissolve the Soviet Union
(b) Republic including Chechnya constitutes the Russian Federation
(c) Ingushetia (the border province) is now formally separate from Chechnya
(d) Lithuania and Latvia did not separate from the Soviet Union
Ans. (d)
Q. 17. Given below are two statements, one labelled as Assertion (A) and the other labelled as Reason (R):
Assertion (A): The distinction between written and unwritten constitution is one of degree rather than of kind.
Reason (R): There is no constitution which is entirely written or entirely unwritten.
In the context of the above two statements, which one of the following is correct?
Codes:
(a) Both A and Rare true and R is the correct explanation of A
(b) Both A and R are true but R is not the correct exp1anationof A
(c) A is true but R is false
(d) A is false but R is true
Ans. (a)
Q. 18. In the American Presidential election, in case no candidate secures required majority, the decision is referred to the:
(a) electorate
(b) electoral college
(c) House of Representatives
(d) Senate
Ans. (c)
Q. 19. Which of the following factors have helped the U.S. Federal Government to become powerful vis--vis the state government?
1. The role and personalities of some American Presidents.
2. Doctrine of Implied-Powers given by the U.S. Supreme Court.
3. Grants-in-aid given by the U.S. Federal Government to the States.
4. Emergency situations viz. World wars and the Great Depression.
Codes:
(a) only 1
(b) l and 2
(c) 1, 2 and 3
(d) 1, 2, 3 and 4
Ans. (d)
Q. 20. In the U.S.A the filibuster is a method used by:
(a) the Congress to pressurize the President
(b) the President to prevent the introduction of a Bill in the Congress
(c) members of the Senate to obstruct the passage of a Bill
(d) judiciary to prevent Congress from passing a Bill
Ans. (c)
Q. 21. The relationship between the government and the President in France is a complex one due to:
(a) direct election of the President
(b) centralization of powers
(c) a mixed presidential parliamentary form of government
(d) parliaments reduced powers to control the government
Ans. (c)
Q. 22. The constructive no-confidence vote in the Basic Law of Germany means that the Bundestag can overthrow a Chancellor:
(a) only by special majority vote
(b) only if it is first able to agree on a successor
(c) only with the consent of the President
(d) only with the consent of the Federal Constitution Court
Ans. (b)

Muftbooks.com

MCQs for political science optional aspirants


Q. 23. The National Parliamentary body of China is known as:
(a) State Council
(b) National Peoples Congress
(c) Peoples Assembly
(d) Supreme Soviet
Ans. (b)
Q. 24. The practice adopted by the party in power to reward its members and supporters through government contracts and jobs is
called:
(a) division of powers
(b) implied powers
(c) power politics
(d) spoils systems
Ans. (d)
Q. 25. Which one of the following is not a common feature of federalism in U.S.A and India?
(a) Distribution of powers between the Union and the States
(b) The existence of the Supreme Court
(c) Two sets of judicial organizations
(d) Written Constitution
Ans. (c)
Q. 26. Which one of the following statements is correct?
(a) According to the Constitution of France, the Cabinet consists to the Constitution of France, the Cabinet consists of the President, the
Prime Minister and other minister
(b) The President of France determines the policies of the nation and is responsible to the Parliament
(c) The President of France presides over meetings of the Council of Ministers but the Prime Minister is responsible for the policies of
the nation
(d) The Prime Minister of France neither expected to determine the composition of the Cabinet nor he presides over its meetings
Ans. (c)
Q. 27. The German Chancellor is elected
(a) directly by the people
(b) by the Bundesrat (Upper House)
(c) by the Bundestag (House of Representative)
(d) by both the House (Bundesrat and Bundestag)
Ans. (c)
Q. 28. Which one of the following statements is not correct in respect of China?
(a) The cultural revolution (1964-1974) was not a success
(b) The Communist Party of China is organized on the basis of democratic centralism
(c) The Chinese constitution provides for a uniform judicial system
(d) Maos ideas of New Democracy have been put into practice in China
Ans. (a)
Q. 29. Which one of the following is not correct in respect of China?
(a) The Communist Party of China is organized on the basis of democratic centralism
(b) The Chinese constitution provides for a uniform judicial system
(c) Maos ideas of New Democracy have been put into practice
(d) The Chinese Constitution provides for a bicameral legislature
Ans. (d)
Q. 30. Which one of the following is not a feature of French Constitution of 1958?
(a) Adoption of Declaration of Rights of 1789
(b) Favoring strong Presidency
(c) Limited powers of Parliament
(d) Unicameral legislature
Ans. (d)
Q. 31. Which Constitution is called a tailor-made Constitution?
(a) Constitution of Fourth Republic of France
(b) Constitution of Fifth Republic of France
(c) Constitution of United Germany
(d) Constitution of U.S.A
Ans. (b)
Q. 32. Consider the following statements about the Constitution of U.K:
1. U.K. has a unitary Constitution
2. Parliament is sovereign in U.K.
3, U.K. has a system of judicial review
Which of the statements given above is/are correct?
(a) 1, 2 and 3
(b) 2 and 3
(c) 1 and2
(d) l only
Ans. (c)

Muftbooks.com

MCQs for political science optional aspirants


Q. 33. Which of the following British Prime Minister proposed the abolition of judicial powers of the House of Lords?
(a) Margaret Thatcher
(b) John Major
(c) Tony Blair
(d) Harold Wilson
Ans. (c)
Q. 34. Which one of the following is correct with regard to the impeachment of the President of the USA?
(a) President can be impeached only through a referendum
(b) President can be impeached only the Senate
(c) Impeachment process is initiated in the House of Representatives and the Senate acts as trial chamber
(d) Proposal for impeachment is init4ated in the Senate and the House of Representatives acts as a trial chamber
Ans. (c)
Q. 35. Which one of the following is not a part of the American constitutional practice?
(a) Pocket veto
(b) Collective responsibility
(c) Senatorial courtesy
(d) Filibustering
Ans (b)
Q. 36. Match List- I (Countries) with List- II (Types of Political System) and select the correct answer using the codes given below the
Lists:
List -I List- II
A. China 1. Federal, Representative, Democratic, Republic
B. Germany 2. Sovereign, Democratic, Republic,
C. France 3. Peoples Democratic, Socialist, Republic
D. South Africa 4. Indivisible, Secular, Democratic, Republic
Codes:
ABCD
(a) 4 2 3 1
(b) 3 1 4 2
(c) 4 1 3 2
(d) 3 2 4 1
Ans. (b)
Q. 37. The responsibility of interpreting and supervision of the implementation of the Constitution of China rests with which one of the
following?
(a) Standing Committee of National Peoples Congress
(b) Supreme Peoples Court
(c) Central Military Commission
(d) State Council
Ans. (a)
Q. 38. The idea that before passing a no confidence motion against the government, the Parliament must explore the possibility of
forming an alternative government is a part of:
(a) Constitution of France
(b) Constitution of US
(c) Constitution of the Republic of Ireland
(d) Constitution of Germany
Ans. (d)
Q. 39. Which one of the following countries was described by China as a Social imperialist power?
(a) U.S.A
(b) U.S.S.R
(c) India
(d) Japan
Ans. (b)
Q. 40. The Institution of Ombudsman was first introduced in:
(a) New Zealand
(b) Norway
(c) Finland
(d) Sweden
Ans. (d)
Q. 41. Which one of the following statements describes a unitary system?
(a) Where the territorial divisions are subordinate agencies of the Central Government.
(b) Where the territorial divisions have coordinate status with the central Government.
(c) A device to reconcile national unity with rights of the units.
(d) Where the functions of the government are vested ma single individual.
Ans. (a)
Q. 42. Which of the following events led to the establishment of the Fifth Republic in France?
(a) Algerian crisis
(b) Suez crisis

Muftbooks.com

MCQs for political science optional aspirants


(c) Cuban crisis
(d) Macau crisis
Ans. (a)
Q. 43. In the US Presidential election, in the event of no candidate securing the required majority, with whom is the decision left?
(a) Electorate
(b) Electorate college
(c) House of Representatives.
(d) Senate
Ans. (c)
Q. 44. Consider the following statements:
1. The American Congress is a sovereign legislature in the sense the British Parliaments is.
2. Bicameralism found new foundation when the framers of the American constitution designed it in the Philadelphia Convention in
1787.
Which of the statement given above is/are correct?
(a) 1 only
(b) 2 only
(c) Both 1 and 2
(d) Neither 1 nor 2
Ans. (b)
Q. 45. Consider the following statements in respect of the Constitution of Germany:
1. The number of seats in the Bundestag varies and the numbers of the Bundestag are elected for a term of five years.
2. The Federal President is elected for a five year term by the Bundestag.
Which of the statements given above is/are correct?
(a) 1 only
(b) 2 only
(c) Both 1 and 2
(d) Neither 1 nor 2
Ans. (b)
Q. 46. Consider the following statements in respect of the constitution of South Africa.
1. The Constitution of the Republic of South Africa was amended to provide that the constitutional Court Judges are appointed for a
non-renewable 10 year term of office or until they reach the age of 70 years.
2. If a bill is rejected by one house then it is referred back o both the house i.e. the National Assembly and the National council of
Provinces after consideration by the Mediation Committee.
Which of the statements given above is/are correct?
(a) 1 only
(b) 2 only
(c) Both 1 and 2
(d) Neither 1 nor 2
Ans. (c)
Q. 47. Consider the following statements in respect of the Constitution of China.
1. National Peoples Congress can amend the constitution and is elected for a period of 5 years.
2. The State Council is the Supreme executive organ
and comprises the Prime Minister, Deputy Prime Ministers and state Councilors.
Which of the Statements given above is/are correct?
(a) l only
(b) 2only
(c) Both 1 and 2
(d) Neither 1 nor 2
Ans. (c)
Q. 48. What is the reason for the complexity of the. relationship. between the government and the President in France?
(a) Direct election of the President
(b) Centralization of powers
(c) Combination of the Presidential and the Parliamentary forms of Government
(d) Parliaments reduced powers to control the government.
Ans. (c)
Q. 49. Which among the following was one of the important factors that brought down the authoritarian regime in Poland?
(a) Collapse of the Berlin Wall
(b) Growth of Global Civil Society
(c) Growth of Solidarity Movement
(d) Retirement of Gorbachev
Ans. (b)
Q. 50. Consider the following statements in respect of the British Civil Services.
1. Northcote Trevelyan report recommended for merit system of recruitment and opposed the generalist cadre of British top civil
posts.
2. Fulton committee stressed the need for generalist service and was not in favour of induction of specialists into British civil service.
Which of the statements given above is/are correct?
(a) 1 only
(b) 2 only

Muftbooks.com

MCQs for political science optional aspirants


(c) Both 1 and 2
(d) Neither 1 nor 2
Ans. (c)
.....................................-------------------------------------..................................................................
Objective Political Science
POLITICAL SCIENCE SOLVED OBJECTIVE PAPER
Sample paper MCQ model test objective questions political Science.
political science for upsc
Q. 1. Manipulation of boundaries to enable the party in power to capture (at the cost of other parties) as many seats as possible, has
come to be known as:
(a) filibustering
(c) guillotine
(b) gerrymandering
(d) log-rolling
Ans. (b)
Q. 2. Which one of the following countries introduced universal adult franchise is one move?
(a) U.K.
(b) India
(c) U.S.A.
(d) Australia.
Ans. (b)
Q. 3. Who among the following argued that universal suffrage does injustice to property owners?
(a) Henry Maine
(b) H.J. Laski
(c) J.S. Mill
(d) W. F. Willoughby
Ans. (c)
Q. 4. Which one of the following is the judicial function of the executive?
(a) Passing bills
(b) Protecting legis1atie privileges
(c) Impeachment
(d) Giving reprieve to persons who have been awarded capital punishment
Ans. (d)
Q. 5. Public opinion necessarily means opinion.
(a) based on reason
(b) of the majority
(c) which aims at the welfare of the whole community

Muftbooks.com

MCQs for political science optional aspirants


(d) of all concerned
Ans. (c)
Q. 6. J.S. Mill recommended complete freedom of speech and censorship:
(a) in the case of a false view
(b) in the cases of a false view and a view that is partially true and partially false
(c) in the case of view that is both.-partially true and partially false
(d) in all cases irrespective of whether a view is partially or wholly true or false
Ans. (d)
Q. 7. Who among the following was of the view that control of a corporate world on mass media has transformed the press from a
watchdog of democracy to a system of thought control for duping ordinary citizens into conforming to corporatism?
(a) Noam Chomsky
(b) Raymond Aron
(c) John Rawls
(d) Karl Popper
Ans. (b)
Q. 8. Who among the following described democracy as the tyranny of the majority?
(a) J. Rousseau
(b) De Tocqueville
(c) John Dunning
(d) James Madison
Ans. (b)
Q. 9. The thinker who has been characterized as a reluctant democrat is:
(a) J. Bentham
(b) E. Burke
(c) T.H. Green
(d) J.S. Mill
Ans. (d)
Q. 10. Which one of the following functions pertains only to political party and not to pressure group?
(a) Collecting money for the organization
(b) Contesting political election with own symbol
(c) Organizing public meetings and rallies
(d) Publishing pamphlets and leaflets
Ans. (b)
Q. 11. Who among the following is not an exponent of Elitist Theory of Democracy?

Muftbooks.com

MCQs for political science optional aspirants


(a) Michel
(b) Pareto
(c) Mosca
(d) C. Macpherson
Ans. (d)
Q. 12. Who among the following saw parties/candidates as motivated solely by desire to win elections and tending to converge towards
the views of the median vote in two party plurality based competition?
(a) Anthony Downs
(b) Robert Dahl
(c) 3. Coleman
(d) D.A. Wittman
Ants. (a)
Q. 13. Given below are two statements, one labelled as Assertion (A): and the other labelled as Reason (R):
Assertion (A): Public opinion thrives in a democratic society.
Reason (R): There is freedom of the press.
In the context of the above two statements, which one of the following is correct?
Codes:
(a) Both A and R are true and R is the correct explanation of A
(b) Both A and R are true but R is not the correct explanation of A
(c) A is true but R is false
(d) A is false but R true
Ans. (a)
Q. 14. Who among the following advocated partyless democracy in India?
(a) Mahatma Gandhi
(b) M. N. Roy
(c) Jayprakash Narayan
(d) Acharya Vinoba Bhave
Ans. (c)
Q. 15. Given below are two statements, one labelled as Assertion (A) and the other labelled as Reason (R):
Assertion (A): While pressure groups perform the task of interest articulation, political parties perform the function of interest
aggregation.
Reason (R): Pressure groups represent homogeneous. interest groups and seek political power.
In the context of the above two statements, which one of the following is correct?
(a) Both A and R are true and R is the correct explanation of A
(b) Both A and R are true but R is not the correct explanation of A

Muftbooks.com

MCQs for political science optional aspirants


(c) A is true but R is false
(d) A is false but R is true
Ans. (b)
Q. 16. Assertion (A): Oligarchy and Democracy as systems of governments can be clearly distinguished.
Reason (R): According to Elite Theory, an oligarchy exists behind the democratic facade.
Codes:
(a) Both A and Rare true and R is the correct explanation of A
(b) Both A and R are true but R is not the correct explanation of A
(c) A is true but R is false
(d) A is false but R is true
Ans. (b)
Q. 17. Who among the following theorists advocated participatory democracy?
(a) F. A. Hayek
(b) C.B. Macpherson
(c) Michael Oakeshoot
(d) Gaetano Mosca
Ans. (b)
Q. 18. Who among the following has described democracy as the institutional arrangement for arriving at political decisions in which
individuals acquire the power to decide by means of a competitive struggle for the Peoples vote?
(a) Robert Dahl
(b) Peter Bacharach
(c) C. Wright Mills
(d) Joseph Schumpeter
Ans. (d)
Q. 19. Who among the following has described the world wide triumph of liberal democracy as the end of history?
(a) Samir Amin
(b) David held
(c) Francis Fukuyama
(d) Samuel P. Huntington
Ans. (c)
Q. 20. In which one of the following countries, did women get the right to vote only in 1971?
(a) Australia
(b) New Zealand
(c) Mexico

Muftbooks.com

MCQs for political science optional aspirants


(d) Switzerland
Ans. (d)
Q. 21. Which one of the following is not the reason for the rejection of liberal democracy by Marxists?
(a) Piecemeal reform of Capitalism is not possible
(b) Popular participation in liberal democracy is limited
(c) Unelected and unaccountable centers of power in liberal democracies
(d) In liberal democracy, the government is limited
Ans. (a)
Q. 22. D.D McKean has described pressure groups as:
(a) Anonymous Empire
(b) Freedom Flowers
(c) Interest Articulations
(d) Invisible Government
Ans. (d)
Q. 23. A close working relationship between interest groups and the administrative agencies; which are centers of policy making and
policy implementation, is known as:
(a) institutional interest group
(b) associated interest group
(c) business interest group
(d) clientele interest group
Ans. (a)
Q. 24. Indian National Trade Union Congress is an example of an:
(a) associational pressure group
(b) institutional pressure group
(c) ideological group
(d) anomie pressure group of an associational pressure group. It includes all traders, labourers, farmers unions.
Ans. (d)
Q. 25. Who among the following favoured qualification as the basis for right to vote?
(a) J. Bentham
(b) J.S. Mill
(c) T.H. Green
(d) E. Barker
Ans. (b)
Q. 26. Participatory democracy calls for:

Muftbooks.com

MCQs for political science optional aspirants


(a) greater involvement of the legislature in the business of legislature
(b) active engagement of the representatives in the affairs of their constituencies
(c) greater and active engagement of citizens in government
(d) increasing the voter turnout in elections
Ans. (c)
Q. 27. Which one of the following is not a mechanism to ensure political accountability?
(a) Periodic elections
(b) Informed public opinion
(c) Free press
(d) Pressure groups
Ans. (d)
Q. 28. Most favoured technique of pressure groups, in the U.S.A is:
(a) boycott and picketing
(b) lobbying
(c) total strike
(d) peaceful agitations
Ans. (b)
Q. 29. Match List- I (Group) with List -II (Concern / Distinctiveness) and select the correct answer:
List- I

List- II

A. NGOs

1. To promote interests of their members and to influence politics

B. Pressure groups

2. A group of eminent persons in their chosen field.

C. Political parties
D. Elites

3. Concerned with some public issues/problems


4. Seek political power

Codes:
ABCD
(a) 3 1 4 2
(b) 4 2 3 1
(c) 4 1 3 2
(d) 3 2 4 1
Ans. (a)
Q. 30. Given below are two statements, one labelled as Assertion (A) d the other labelled as Reason (R):
Assertion (A): Liberal democracy constitution is the basis of the growing internationalism today.
Reason (R): After the disintegration of that Soviet Union and Eastern Europe there is no effective rival to liberal democracy in the
worlds

Muftbooks.com

MCQs for political science optional aspirants


In the context of the above two statements, which one of the following is correct?
Codes:
(a) Both A and R are true and R is the correct explanation of A
(b) Both A and R are true but R is not the correct explanation of A
(c) A is true but R is false
(d) A is false but R is true
Ans. (a)
Q. 31. Democracy is desirable because the aggregate satisfaction it offers is better than that in other systems of rue, is the argument
forwarded by:
(a) liberals
(b) utilitarian
(c) marxists
(d) socialists
Ans. (b)
Q. 32. Among the following identify the most powerful and largest number of pressure groups in India:
(a) Institutional groups
(b) Public interest groups
(c) Non-associational groups
(d) Associational groups
Ans. (a)
Q. 33. Role of the pressure group is confined to the welfare of:
(a) all sections of society within a state
(b) its members only
(c) mankind
(d) weaker sections of society
Ans. (b)
Q. 34. Who among the following is an acclaimed analyst on political parties?
(a) Walter-Lippmann
(b) Maurice Duverger
(c) H. J. Laski
(d) James Bryce
Ans. (b)
Q. 35. Given below are two statements, one labelled as Assertion (A) and the other labelled as Reason (R):
Assertion (A): Pressure Groups involve in politics and policy making process in India through direct and indirect means

Muftbooks.com

MCQs for political science optional aspirants


Reason (R): They aspire to come to power to achieve their goals.
In the context of the above two statements, which one of the following is correct?
Codes:
(a) Both A and R are individually true and R is the correct explanation of A
(b) Both A and R are individually true but R is not the correct explanation of A
(c) A is true but R is false
(d) A is false but R is true
Ans. (c)
Q. 36. A partyess regime is a conservative regime an anti-party regime is a reactionary regime. Who made this statement?
(a) K.C. Wheare
(b) Carl J. Friedrich
(c) Sartori
(d) Samuel Huntington
Ans. (c)
Q. 37. The advocates of deliberative democracy emphasize
(a) parliamentary sovereignty
(b) judicial autonomy
(c) executive supremacy
(d) popular participation
Ans. (d)
Q. 38. S. Finer has designated the interest groups as:
(a) invisible governments
(b) unofficial governments
(c) Government within the governments
(d) anonymous empire
Ans (d)
Q. 39. Democracy is an institutional arrangement for arriving at political decisions and not an end in itself. This argument is put forth
by:
Scholars

Books

(a) Joseph Schumpeter

1. Capitalism, Socialism and Democracy

(b) Carole Pateman

2. Participation and Democratic Theory

(c) Milton Friedman

3. Capitalism and Freedom

(d) Robert Nozick

4. Anarchy, State and Utopia

Ans. (a)

Muftbooks.com

MCQs for political science optional aspirants


Q. 40. Which of the following procedures is/are used in the implementation, of Minority Representation under the scheme of
Proportional Representation?
1. Hare scheme of single transferable vote
2. List system
3. Limited vote plan
4. Commutative vote system
Select the correct answer using the codes given below:
Codes:
(a) 1 only
(b) l and 2
(c) l and 3
(d) 3 and 4
Ans. (b)
Q. 41 Consider the following statements:
The major functions of political parties are:
1. rulemaking
2. interest aggregation
3. political socialization
4. regulation of the behaviour of the people
Which of these statements are correct?
(a) 1 and 2
(b) 2 and 3
(c) 3 and 4
(d) 1 and 4
Ans. (b)
Q. 42. Which one of the following is not a characteristic of Liberal Democracy?
(a) Guarantees of civil liberties and rights
(b) Government is vested with unfettered powers
(c) The independence of organized groups and interests from government
(d) Regular elections that respect the principle of one person one vote; one vote one value
Ans. (b)
Q. 43. Alternative vote of confidence is in vogue in one of the democratic political system, identify the country:
(a) Switzerland
(b) Austria

Muftbooks.com

MCQs for political science optional aspirants


(c) Germany
(d) France
Ans. (c)
Q. 44. A technique commonly used by the pressure groups in the USA is:
(a) boycott and picketing
(b) lobbying
(c) lawsuits
(d) peaceful agitations
Ans. (b)
Q. 45. Who among the following advocated partyless democracy in India?
(a) M. N. Roy
(b) Jayprakash .Narayan
(c) Acharya Narender Dev
(d) Ram Manohar Lohia
Ans. (b)
Q. 46. Which one of the following is the following is the primary purpose of a pressure group?
(a) To criticize the government
(b) To contest the elections
(c) To formulate policy
(d) To bring pressure on government to influence policy decision
Ans. (d)
Q. 47. Participatory democracy seeks to enhance which one of the following?
(a) Decentralization
(b) Representation of disadvantaged sections
(c) Active accountability
(d) Political accountability
Ans. (c)
Q. 48. Given below are two statements, one labelled as Assertion (A) and the other labelled as Reason (R):
Assertion (A): Democratic majorities can easily decide not to respect particular rights, especially those of unpopular minorities.
Reason (R): Democracy must respect some basic right, namely those necessary for the effective exercise of democratic citizenship.
In the context of the above two statements, which one of the following is correct?
Codes:
(a) Both A and R are individually true and R is the correct explanation of A
(b) Both A and Rare individually true but R is not the correct explanation of A

Muftbooks.com

MCQs for political science optional aspirants


(c) A is true but R is false
(d) A is false but R is true
Ans. (b)
Q. 49. Given below are two statements, one labelled as Assertion (A) arid the other labelled as Reason (R):
Assertion (A): In a multiparty electoral system, through a pre-election coalition with smaller parties, major party enhance the
possibility of winning more seats then it would have own on its own.
Reason (R): In a multiparty system there is no necessary correspondence between the votes received and the seats won by a political
party. In the context of the above two statements, which one of the following is correct?
Codes:
(a) Both A and R are individually true and R is the correct explanation of A
(b) Both A and Rare individually true but R is not the correct explanation of A
(c) A is true but R is false
(d) A is false but R is true
Ans. (b)
Q. 50. Regular elections are:
(a) the core value of democracy
(b) not held in communist countries
(c) a necessary but not a sufficient condition of democracy
(d) used by military rulers to gain legitimacy
Ans. (c)
..............................................----------------------------------------...........................................
ugc net pol science solved model questions
solved model question papers for UGC CSIR Political Science
ugc net Pol science solved sample paper
UGC NET Political Science Model question papers
Q. 1. Which one of m4in.com.ebozavr.com the following pairs is not correctly matched ?
(a) Positivism

: Leo Strauss

(b) Post Modernism

: Jacques Derrida

(c) Feminism

: Simone de l3eauvoire

(d) Post-Structuralism

: Michael Foucault

Ans. (a)
Q. 2. The libertarians among the neo-liberals are opposed to:
(a) rolling back the state
(b) slimming the bureaucracy
(c) restriction on monopolies
(d) freedom of the market
Ans. (a)

Muftbooks.com

MCQs for political science optional aspirants


Q. 3. Karl Marx and F. Engels co-authored:
(a) Das Captial
(b) Anti-Duhring
(c) German Ideology
(d) Critique of the Gothe Programme.
Ans. (c)
Q. 4. Which of the following pairs is not correctly matched?
(a) Karl Marx

: The Poverty of Philosophy

(b) V.1. Lenin

: Sate and Revolution

(c) Josef Stalin

: Wage, Labour and Capital

(d) Friedrich Engels

: Origin of Family, Private Property and the State

Ans. (c)
Q.5. Match List- I (Theorists) with List- II (Theories) and select the correct answer
List-I

List II

A. Sydney Webb

1. Syndicalism

B. Georges Sorel

2. Fabian Socialism

C. Saint Simon

3. Utopian Socialism

Edward Bernstein

4. Revisionism

Codes:
ABCD
(a) 2 1 3 4
(b) 2 1 4 3
(c) 1 2 3 4
(d) 1 2 4 3
Ans. (a)
Q. 6. Which one of the following principles, did fascism not subscribe to?
(a) Corporatist understanding of society
(b) Racial superiority
(c) Dictatorship of the proletariat
(d) Obedience and discipline
Ans. (c)
Q. 7. Means are after all everything. As the means, so the end; there is no wall of separation between means and end.
The above statements are characteristic of:
(a) M.K. Gandhi

Muftbooks.com

MCQs for political science optional aspirants


(b) B.R. Ambedkar
(c) Mao-Zedon
(d) Karl Marx
Ans. (a)
Q. 8. According to M.K. Gandhi, the five eternal guides of human conduct are:
(a) Ahimsa, Satya, Asteya, Aparigraha and Brahmacharya
(b) Ahimsa, Satya, Vinaya, Daya and Tapas
(c) Ahimsa, Karuna, Shila, Bhakti and Karma
(d) Satya, Dharma, Artha, kama and Tapas
Ans (a)
Q. 9. Neibuhr used the term Children of darkness for the advocates of:
(a) idealism
(b) non-liberalism
(c) marxism
(d) realism
Ans. (c)
Q. 10. Match List -I with List- II from the points of view of resemblance of Political ideas and select the correct answer:
List I
(Indian Thinkers)

List-II
(Political Thinkers)

A. M.K. Gandhi

1. John Dewey

B. Jawaharlal Nehru

2. V.I. Lenin

C. B.R. Ambedkar

3. John Ruskin

D. E.M.S. Namboodripad

4. Sidney Webb

Codes:
ABCD
(a) 3 1 4 2
(b) 2 4 1 3
(c) 2 1 4 3
(d) 3 4 1 2
Ans. (d)
Q. 11. Given below are two statements, one labelled as Assertion (A): and the other labelled as Reason (R):
Assertion (A) : Through Satyagraha and Ahimsa, Gandhi not merely attempted to transform the external world but also sought
freedom from inner passions and impulses
Reason (R): Gandhis concept of Swaraj meant not merely freedom from external constraints but also self-realization.

Muftbooks.com

MCQs for political science optional aspirants


In the context of the above two statements, which one of the following is correct?
Codes:
(a) Both A and R are true and R is the correct. explanation of A
(b) Both A and R are true but. R is not the correct explanation of A
(c) A is true but R is false
(d) A is false but R is true
Ans. (a)
Q. 12. Given below are two statements, one labelled as Assertion (A) and the other labelled as Reason (R):
Assertion (A) : During the inter-war period, liberal parties all over Europe became strong.Reason (R): The liberal principles were
accepted by the various democratic parties.
In the context of the above two statements, which one of the following is correct?
Codes:
(a) Both A and R are true and R is the correct explanation of A
(b) Both A and R are true but R is not the correct explanation of A
(c) A is true but R is false
(d) A is false but R is true
Ans. (d)
Q. 13. Given below are two statements, one labelled as Assertion (A) and the other labelled as Reason (R):
Assertion (A) : Marxists. are skeptical about that likelihood of an equitable social order following from interdependence and
transationaism.
Reason (R) : Marxists believe that dominant classes f the core states would use their power to control those process ensuring that
interdependence remains highly asymmetrical and that transnational institutions and regimes reinforce the capitalist
system.
In the context of the above two statements, which one of the following is correct?
Codes:
(a) Both A and R are true and R is the correct explanation of A
(b) Both A and R are true but R is riot the correct explanation of A
(c) A is true but R is false
(d) A is false but R is true
Ans. (a)
Q. 14. The Chief modus operandi of cultural imperialism is:
(a) annexation
(b) plunder
(c) trade
(d) hegemony

Muftbooks.com

MCQs for political science optional aspirants


Ans (d)
Q. 15. The most outstanding contemporary advocate, of libertarianism is:
(a) Rawls
(b) Habermas
(c) Dworkin
(d) Nozick
Ans. (d)
Q. 16. Match List- I with List- II and select the correct answer:
List- I

List- II

(Phrase)
A. Hayes

1. Of all the evils I hate, I think I have nationalism most.

B. Organsko

2. An organized self interest of the whole people

C. Rabindranath Tagore

3. As a belief nationalism has been a curse and nothing but a

curse
D. Victor Golanez

4. To believe that nationalism will vanish because it is

dangerous or the nations will wither away because


there are other ways of organizing mankind is false.
Codes:
ABCD
(a) 1 2 4 3
(b) 3 4 2 1
(c) 1 4 2 3
(d) 3 2 4 1
Ans. (d)
Q. 17. Marxism believes that:
(a) the real is the rational and the rational is the real
(b) social relations are rooted in the material conditions
(c) economic relations can be understood in the light of the progress of human mind
(d) it is human consciousness that determines the social life of man
Ans. (b)
Q. 18. Match List -I with List- II and select the correct answer:
List- I

List-II

(Characteristic Features) (Political Ideology)


A. Formation of the natural harmonious,

1. Guild socialism

Muftbooks.com

MCQs for political science optional aspirants


organic communities for the free
development of the individual and
the abolition of the main source of
coercionstate
B. Formation of self- governing autonomous

2. Democratic socialism

association of, people that will work on


cooperative basis to establish partnership
between groups and State.
C. Combination of human fellowship just

3. Syndicalism

social system, common ownership and


mutual service
D. A system of revolutionary tactics using

4. Anarchism

non-political methods to create an


economic federation
Codes:
ABCD
(a) 4 1 2 3
(b) 2 1 4 3
(c) 4 3 2 1
(d) 2 3 4 1
Ans. (a)
Q. 19. Everything for the State, nothing against the State, nothing outside the State sums up the philosophy of:
(a) Marxism
(b) Fascism
(c) Fabian Socialism
(d) Guild Socialism
Ans. (b)
Q. 20. Which one of the following is not the characteristic of a six point syndrome of totalitarianism propounded by Friedrich and
Brzezinski?
(a) An official ideology
(b) A system of terroristic policy
(c) A monopoly of means of communication
(d) A divine right to rule
Ans. (b)

Muftbooks.com

MCQs for political science optional aspirants


Q. 21. Consider the following statements:
Classical liberalism stands for:
1. a state which is merely a watchman
2. individualism
3. welfarism
4. free market
Which of these statements are correct?
(a) 1 and 2
(b) 1, 2 and 3
(c) 3 and 4
(d) 1, 2 and 4
Ans. (d)
Q. 22 Consider the following statements:
1. Authority that does not accept freedom is authoritarianism
2. Authoritarianism does not distinguish between authoritative personality and authoritarian personality
3. Authoritarianism is directive in its orientation and is disregardful of public opinion
4. Authoritarianism thinks that it knows what is good for the citizens.
Which of the above statements associated with authoritarianism are correct?
(a) l and 2
(b) 2 and 3
(c) 1, 3 and 4
(d) l, 2, 3 and 4
Ans. (d)
Q. 23. Which one of the following is not a statement of Karl Marx?
(a) The State is an executive committee for managing the affairs of the bourgeoisie
(b) Capitalism digs its own grave
(c) Imperialism is the highest state of capitalism
(d) The state will wither away
Ans. (c)
Q. 24. Consider the following statements:
Marxism is critical of liberalism for its:
1. narrow concern with civic and political rights
2. legitimization of capitalist class relations
3. inability to perceive surplus value generated in

Muftbooks.com

MCQs for political science optional aspirants


capitalist production system
Which of the statements given above are correct?
(a) 1, 2 and 3
(b) 1 and 2
(c) 2 and 3
(d) 1 and 3
Ans. (b
Q. 25. In Marxs view, the social revolution beings when relations of production become a fetter upon the further development of forces
of production.
The relations of production, in this context, refer to:
(a) relations between the nations
(b) relations between surplus value and absolute surplus value
(c) relation between production and distribution
(d) relation between the basic classes
Ans. (d)
Q. 26. Democratic socialism distinguishes itself from scientific socialism by which of the following attributes?
1. Rejection of market mechanism to decide on production and distribution policies.
2. Acceptance of central planning
3. Insistence on democratic methods of decision- making
Select the correct answer using the codes given below:
Codes:
(a) 1, 2 and 3
(b) 1 only
(c) 2 and 3
(d) 3 only
Ans. (d)
Q. 27. Gandhi reasoned that the primary responsibility for the removal of the evil of untouchabiity in lndia rests on:
(a) Caste Hindus
(b) Indian nation
(c) untouchables themselves
(d) Indian National leadership
Ans. (a)
Q. 28. Consider the following statements about rationalism:
1. Rationalism represents a philosophical doctrine that the world is known by reason

Muftbooks.com

MCQs for political science optional aspirants


2. Rationalism represents the pursuit of value free science
3. Rationalism represents the solutions in place of religious beliefs
4. Rationalism studies reasons for social actions.
Which of the statements given above are correct?
(a) 1 and 3
(b) 1, 3 and 4
(c) 2 and 4
(d) 2, 3 and 4
Ans. (b)
Q. 29. With whom of the following is conservation, as a political ideology associated?
1. Burke
2. Hobbes
3. Hegel
4. Oakeshoot
Select the correct answer using the codes given below:
Codes:
(a) 1, 2 and 3
(b) 2 and 4
(c) 1 and 3
(d) 1, 3 and 4
Ans. (d)
Q. 30. The distinction between the private sphere of family and the public sphere of politics is central to which one of the following
trends of thought?
(a) Athenian (Greek) philosophy
(b) Liberalism
(c) Marxism
(d) Gandhism
Ans. (a)
Q. 31. Cultural nationalism is a form of nationalism that places primary emphasis on the regeneration of the nation as:
(a) a discrete political community
(b) a distinctive civilization
(c) a modern State
(d) an artificial entity
Ans. (b)

Muftbooks.com

MCQs for political science optional aspirants


Q. 32. According to Quincy Wright, which one of the following indicates the correct order of successive stages of nationalism?
(a) Medieval Monarchical Revolutionary Liberal Totalitarian
(b) Medieval Revolutionary Monarchical Totalitarian Liberal
(c) Medieval Monarchical Totalitarian Revolutionary Liberal
(d) Monarchical Medieval Revolutionary Totalitarian Liberal
Ans (a)
Q. 33. For Gandhi, Swaraj meant:
(a) selfrule
(b) rule of law
(c) rule by morally good persons
(d) pursuit of good of all
Ans. (d)
Q. 34. Which one of the following is not a tenet of Stalinism?
(a) The theory of permanent revo1utio
(b) Socialism in one country
(c) Intensification of class struggle with socialist advance
(d) Strong State under socialism
Ans. (a)
Q. 35. Political realism maintains autonomy of:
(a) social sphere
(b) political sphere
(c) national sphere
(d) international sphere
Ans. (b)
Q. 36. Match List- I with List- II and select the correct answer using the code given below the Lists:
List I
A. Dictatorship of the proletariat

List- II
1. Liberalism

B. Avowal of rights

2. Communism

C. Glorification of state

3. Anarchism

D. Opposition to state in general

4. Fascism

Codes:
ABCD
(a) 2 4 1 3
(b) 3 1 4 2

Muftbooks.com

MCQs for political science optional aspirants


(c) 2 1 4 3
(d) 3 2 1 2
Ans. (c)
Q.37. Who among the following thinkers are associated with early socialism sometimes termed as utopian socialism?
(a) Robert Owen
(b) Saint Simon
(c) Charles Fourier
(d) Proudhon
Select the correct answer using the code given below:
(a) 1, 2 and 3
(b) 1, 2, and 4
(c) 3 and 4
(d) 1, 2, 3 and 4
Ans. (a)
Q. 38. Consider the following statements:
1. Fascism can be described as a protagonist of a radical totalitarian state.
2. Hitler and Mussolini tried to transcend class-conflict. in the name of greater and higher ideal of nationalism.
3. Fascism accepted capitalism but did not stress its subordination to the ideal of welfare state.
Which of the statements given above is/are correct?
(a) 1 and 2
(b) 1, 2 and 3
(c) 3 only
(d) 2 and 3
Ans. (b)
Q. 39. Who among the following propagated the idea of Cultural Revolution under Socialism?
(a) Karl Marx
(b) Mao-Tse-Tung
(c) V.I. Lenin
(d) J. Stalin
Ans. (b)
Q. 40. As per Marxism, which one of the following serves as the force, spurring social developing?
(a) Materialism
(b) The Communist Party
(c) Socialist ideology

Muftbooks.com

MCQs for political science optional aspirants


(d) Class contradictions.
Ans. (d)
Q. 41. Which one of the following ideas is integral to Marxism?
(a) Materialist dialectics
(b) Priority of the economic factor
(c) Surplus value as mode of appropriation of surplus under capitalism
(d) Concept of class struggle
Ans. (a)
Q. 42. What is the cornerstone of Marxism?
(a) Attack on bourgeois way to life
(b) Economic determinism
(c) Materialist conception of history
(d) A distinctive philosophy of the state
Ans. (b)
Q. 43. On what basis does Marxs dialectic materialism interpret and analyse the social events?
(a) Material Forces
(b) Communal Forces
(c) Political Forces
(d) International milieu
Ans. (a)
Q. 44. Which one of the following thoughts lays stress on rights of recognition and belonging?
(a) Neo-Liberalism
(b) Conservatism
(c) Communitarianism
(d) Democratic socialism
Ans. (c)
Q. 45. According to Burke a representatives function is to:
(a) act as the mouthpiece of his constituents
(b) to exercise his independent judgment in the interest of his own nation whether it agrees with the vies of his constituents or not
(c) to exercise his independent judgment in the interest of his own constituency
(d) to consult his constituents on major issues of national importance and act accordingly Ans. (b)
Q. 46. Which of the following are the devices through which representation for minorities can be secured?
1. Cumulative Vote system
2. First-past-the-post system

Muftbooks.com

MCQs for political science optional aspirants


3. Direct democracy
4. Proportional Representation
(a) l and 2
(b) l and 4
(c) 2 and 3
(d) 3 and 4
Ans. (b)
Q. 47. The realistic theory of democracy as a critique of the classical democracy was formulated by:
(a) Ernest Barker
(b) Harold Laski
(c). Schumpeter
(d) Hannah Arendt
Ans. (c)
Q. 48. The nineteenth century liberals were distrustful of democracy because
(a) it was in conflict with the principle of liberty
(b) it was opposed to hereditary rule
(c) it emphasized the idea of economic justice
(d) it tended to undermine the power of landed aristocracy
Ans. (a)
Q. 49. Which one of the following statements about totalitarian democracy is correct?
(a) Totalitarian democracy is a contradiction in terms
(b) Totalitarian democracy is a form of democracy consistent with mass society
(c) Totalitarian democracy was prevalent in ancient Greece and the Middle Ages
(d) Totalitarian democracy serves best the interest of each and every member of society
Ans. (a)
Q. 50. In India, partyless democracy was first advocated by:
(a) Jayprakash Narayan
(b) M. N. Roy
(c) Vinoba Bhave
(d) Mahatma Gandhi
Ans. (a)
............................................................-------------------------------------------------------...................
Political Science Solved paper UPSC Civil Services prelims
Political Science for UPSC Civil Services Preliminary Examination
PSC Exam :: Prelims solved papers
UPSC Civil Services: Political Science Prelims Papers Free

Muftbooks.com

MCQs for political science optional aspirants


Q. 1. Which one of the following reflects the true nature of Fascist State as propounded by its theorist?
(a) The Fascist State is an organization with purely material aims, including the creation of peaceful conditions of national life
(b) The Fascist State is a night watchman guaranteeing personal safety to its citizens
(c) The Fascist State is a purely political creation, concerned with the moral and material interests of man
(d) The Fascist State is a spiritual and moral entity absolute in nature
Ans. (d)
Q. 2. Which of the following are glorified by the Fascists?
(a) Violence
(b) War
(c) Leader
(d) Individual.
Choose the correct answer from the codes given below:
Codes
(a) 1, 2 and 3
(b) 2, 3 and 4
(c) 1, 3 and 4
(d) 2 and 4
Ans. (a)
Q. 3. Belief in which of the following is cardinal to Totalitarianism?
1. Utility of democratic institutions
2. Moral values, in politics
3. Sovereignty and omnipotence of the state
4. Complete control over the individuals life
(a) 1 and 2
(b) 2 and 3
(c) 3 and 4
(d) 1 and 4
Ans. (a)
Q. 4. Guild socialism is based on the idea of partnership between the producers and the state in the control of industry. This was said
by:
(a) Hobson
(b) G.D.H. Cole
(c) Laidler
(d) Orange
Ans. (b)
Q. 5. Given below are two statements one labelled as Assertion (A) and the other as Reason (R):
Assertion (A): Liberalism is not committee to maintaining a particular economic system. Reason (R): Liberalism seeks to remove
restrictions on individual liberty and equality.
In the context of the above two statements, which one of the following is correct?
Codes:
(a) Both A and .R are true and R is the correct explanation of A
(b) Both A and R are true but R is not a correct explanation of A
(c) A is true but R is false
(d) A is false but R is true
Ans. (b)
Q. 6. Early liberalism stood for:
(a) religious toleration
(b) recognition of the divine of kings
(c) every thing medieval, both in philosophy and in politics
(d) economic equality
Ans. (a)
Q. 7. The state exits merely because crime exists and its principles function, therefore is to protect and restrain, not to foster and
promote.
The above statement best reflects the ideas of:
(a) Liberalism
(b) Socialism
(c) Anarchism
(d) Fascism
Ans. (a)

Muftbooks.com

MCQs for political science optional aspirants


Q. 8. Which of the following is not a correct statement about centralized planning?
(a) Centralized planning ensures efficiency
(b) Centralized planning is planning from below.
(c) Centralized planning accelerates the rate of economic progress
(d) Centralized planning ruins the prospect of free society
Ans. (b)
Q. 9. Which one of the following statement on planning reflects the thinking of Prof. Hayek?
(a) Collective control of private activities of production and exchange
(b) The direction of productive activity by a central authority
(c) The extension of the functions of public authorities organization and resources
(d) Deliberate direction by persons in charge of large resources of economic activity towards chosen ends
Ans. (d)
Q. 10. The similarity between Fabian Socialism and Marxian Socialism lies in the fact that both:
(a) support the materialistic interpretation on history
(b) believe in the theory of class struggle
(c) support common ownership of the basic means of production
(d) believe in the theory of Surplus Value
Ans. (c)
Q. 11. Given below are two statements one labelled as Assertion (A) and the other labelled as Reason (R)
Assertion (A): The individuals right to unlimited accumulation of private property came to be questioned in the nineteenth century
political theories of Europe.
Reason (R): Such right was seen to enhance the evils of an industrial society.
In the context of the above two statements, which one of the following is correct?
Codes:
(a) Both A and R are true and R is the correct explanation of A
(b) Both A and R are true but R is not a correct explanation of A
(c) A is true but R is false
(d) A is false but R is true
Ans. (a)
Q. 12. Which one of the following is least relevant to the welfare view of functions of the state?
(a) Reasonable amount of social and economic equality
(b) Predominance of group interests over priorities of individuals
(c) Policies conducive to equality of opportunity
(d) Minimization of areas of conflict to maximize conciliation
Ans. (b)
Q. 13. Some idealistic socialists of Europe had tried experiment in community life providing for the individuals:
(a) equal property
(b) common property
(c) collective family
(d) rustic simplicity
Ans. (b)
Q. 14. Which one of the following political philosophies has been described by C.E.M. Joad as a hat, which has lost its shape because
everyone wears it?
(a) Socialism
(b) Communism
(c) Anarchism
(d) Marxism -Leninism
Ans. (a)
Q. 15. Which one of the following statements are true in respect,pf Fabianism?
1. It is a method of propaganda designed to disarm middle class suspicions and make them socialists
2. It is to facilitates the infiltration of socialist ideas
3. It is English collectivist socialism
4. It advocated the collective ownership and management of nations land and capital
(a) 1 and 4
(b) 1 and 2
(c) l, 2 and 3
(d) 2, 3 and 4
Ans. (c)

Muftbooks.com

MCQs for political science optional aspirants


Q. 16. Which of the following can be associated Fabianism?
1. Value is the creation of society and not of labour
2. Land should cease to be personal property of the landlord
3. The changes in the society should be brought by revolution
4. The key industries may be owned privately
(a) 1 and 2
(b) 2 and 3
(c) 2 and 4
(d) 1, 2, 3 and 4
Ans. (a)
Q. 17. Match List -I with List- II and select the correct answer:
List -I List -II
A. The purposes of the state are 1. Totalitarianism
identical with those of society
B. The slogan of Liberty, Equality, 2. Guild Socialism
Fraternity, is replaced by Duty
discipline Sacrifice
C. The welfare of the individual must 3. Nazism
contribute to the welfare of the community
D. Economic organization combined with 4. Democratic socialism
peaceful means to bring about social
reconstruction
Codes:
ABCD
(a) 4 3 1 2
(b) 1 3 4 2
(c) 1 4 2 3
(d) 2 3 4 1
Ans. (b)
Q. 18. Which one of the following is not a characteristic of Socialism?
(a) It is a reaction to the social and economic injustice of the capitalist system
(b) It is a revolt against the exploitation of man by man
(c) It is a protest against an economic and social system based on profit
(d) It strives for the abolition of the State
Ans. (d)
Q. 19. Which one of the following pairs giving names of authors. and their books is not correctly matched?
(a) Hitler : Mein Kampf
(b) J.S. Mill : Utilitarianism
(c) Laski : A Grammar of Politics
(d) Lenin : Communist Manifesto
Ans. (d)
Q. 20. Which one of the following is not a characteristic of a totalitarian State?
(a) Regimentation based on an official body of doctrine. covering all aspects of social life
(b) Regimentation based on an official body of doctrine covering all aspects of social life
(c) Exercising exclusive control over media and communications
(d) Allowing critical scholarship and public opinion to thrive
Ans. (d)
Q. 21. Given below are two statements, one labelled as Assertion (A) and the other labelled as Reason (R):
Assertion (A): Guild Socialism aims at placing authority in .the hands of consumers as they are in majority.
Reason (R): Public Authority in representative democracy is constituted on the principle of majority.
In the context of the above two statements, which one of the following is correct?
Codes:
(a) Both A and R are true and R is the correct explanation of A
(b) Both A and R are true but R is not the correct explanation of A
(c) A is true but R is false
(d) A is false but R is true
Ans. (b)

Muftbooks.com

MCQs for political science optional aspirants


Q. 22. Which one of the following pairs is correctly matched?
(a) Fabian Socialism : Revolution
(b) Syndicalism : Parliamentary Democracy
(c) Guild Socialism : Functional Representation
(d) Democratic Socialism : Dictatorship of the Proletariat
Ans. (c)
Q. 23. Guild Socialism aims at:
(a) maximum liberty for the individual
(b) maximum authority for the state
(c) maximum democracy in industries
(d) maximum authority for the ruling party
Ans. (c)
Q. 24. Consider the passage given below:
Disruption of traditional institutions, identifications and loyalties is likely to lead to ambivalent situations. It is possible that some people
may renew their identification with traditions groups whereas others align themselves with new groups and symbols emergent-from
processes of political development. In addition, political development tends to foster group awareness of variety of class, tribe, religion,
clan, language, religion occupation and others.
Which one of the following is the best explanation of the crucial formulation in the above passage?
(a) Political development is not a unilinear process for it involves both growth and decay
(b) Traditional societies succeed in resisting positive aspects of political development
(c) It is impossible for traditional societies to break away from lingering loyalties
(d) Sustenance to traditional loyalties is conductive to political development
Ans. (a)
Q. 25. Match List I with List -II and select the correct answer:
List -I List- II
A. T.H. Green 1. Natural Rights
B. Jeremy Bentham 2. Idealism
C. John Locke 3. Pragmatism
D. John Dewey 4. Utilitarianism
Codes:
ABCD
(a) 2 4 3 1
(b) 4 2 1 3
(c) 2 4 1 3
(d) 4 2 3 1
Ans. (c)
Q. 26. Which one of the following thinkers may .be called both a utopian socialist and the father of the trade union movement in
Britain?
(a) Adam Smith.
(b) G.D.H Cole
(c) Friedrich Engels
(d) Robert Owen
Ans. (d)
Q. 27. Which one of the following pairs is not correct matched?
(a) Proudhon : Philosophy of Poverty
(b) Robert Owen : New Christianity
(c) Charles Fourier : Phalanx.
(d) Badeuf : Conspiracy of the Equals
Ans (b)
Q. 28. Who among the following were members of the Fabian Society established in 1883?
1. Bertrand Russell
2. George Bernard Shaw
3. Herold Laski
4. H.G Wells
Choose the correct answer from the codes given below:
(a) 1, 2 and 4
(b) 1 and 2
(c) l and 3

Muftbooks.com

MCQs for political science optional aspirants


(d) 2 and 4
Ans. (d)
Q. 29. Which of the following were the doctrinal principles of the Fabians?
1. A. method of propaganda, designed to disarm middle class suspicions and to facilitate the infiltration of socialist ideas.
2. Permeation as their favorite maxim by which was meant that its members were free to join other parties.
3. Preaching d practice of protest against compromises
4. Leading a protest movement against all existing authorities
Select the correct answer from the codes given below:
Q. 30. The three sources of Marxs thought are:
(a) German Philosophy, French, Economy and British Socialism
(b) German Philosophy, Russian Anarchism and French Socialism
(c) German Philosophy, British Economics and French
(d) Utopian Socialism, French Revolution and Tsarist autonomy
Ans. (c)
Q. 31. Match List-I with List- II and select the correct answer:
List I List-II
A. Land, Bread, Peace 1. Chinese Revolution
B. Liberty, Equality, Fraternity 2. French Revolution
C. Nationalism, Livelihood, Democracy 3. Russian Revolution
D. No taxation without representation 4. American Revolution
5. Glorious Revolution
Codes:
ABCD
(a) 2 3 1 4
(b) 2 1 4 3
(c) 3 1 2 5
(d) 3 2 1 4
Ans. (d)
Q. 32. The doctrine concerning the changing of circumstances and upbringing forgets that circumstances are changed by men and that
the educator must himself be educated.
The above view was expressed by:
(a) Hobbes
(b) Feurbach
(c) Hegel
(d) Marx
Ans. (b)
Q. 33. The concept of Two Parliaments was enunciated by:
(a) Guild Socialism
(b) Marxian Socialism
(c) Syndicalism
(d) State Socialism
Ans. (a)
Q. 34. Which of the following were the basic principles of democratic socialism?
1. It aimed at the elimination of the evils of the competition, which creates havoc in a capitalist society.
2. It was against all forms of private property
3. It believed in peaceful democratic and constitutional
4. It was against a competitive party system.
Choose the correct answer from the codes given below:
(a) l and 2
(b) 2 and 3
(c) 3 and 4
(d) 1 and 3
Ans. (d)
Q. 35. Read the following two statements:
A : War alone brings up to their highest tension all human energies, and puts the stamp of nobility upon the peoples who have the
courage to meet it.
B : It is not the consciousness of men that determines their existence, but, on the contrary, their social existence determines
consciousness.

Muftbooks.com

MCQs for political science optional aspirants


Which political ideology is reflected in the respective statement?
(a) Fascism in A and Liberalism in B
(b) Fascism in A and Marxism in B
(c) Liberalism in A and Syndicalism in B
(d) Marxism in A and Fascism in B
Ans. (b)
Q. 36. What is the most appropriate identification of the nature of the State if the follo.ing conditions prevail?
1. Leadership claims that instead of freedom of expression and critical enquiries, sentiment and action are what hold people and groups
together.
2. Society and Nations are synonymous. The State becomes the organic structure of the Nation. The State has absolute sovereignty,
moral and legal
3. Responsibility, Discipline and Hierarchy are essential for citizens participations in national life
4. The liberty of citizens depends exclusively on the extent and effectiveness of State force. The legitimacy of citizens rights is in
congruence with the will of the State.
Choose the correct answer from the codes given below:
Codes:
(a) Bolshevism
(b) Dictatorship
(c) Democratic
(d) Fascism
Ans (d)
Q. 37. One of the foremost exponents of the negative view of Liberty in the twentieth century is:
(a) Isaiah Berlen
(b) C.W Macpherson
(c) John Rawls
(d) Charles Taylor
Ans. (a)
Q. 38. Match List- I with List -II and select the correct answer:
List I List II
(Theory) (Nature of State)
A. Classical liberalism 1. Coercive institution
B. Liberal idealism 2. Necessary good
C. Hegelian idealism 3. Necessary evil
D. Marxism 4.Manifestation of all morality and legality
Codes:
ABCD
(a) 2 3 1 4
(b) 3 2 4 1
(c) 3 2 1 4
(d) 2 3 4 1
Ans. (b)
Q. 39. The 20th Century totalitarianism resembles an onion like structure, according to:
(a) Raymond Aron
(b) Karl Popper
(b) Hannah Arendt
(d) Michael Oakeshoot
Ans. (c)
Q. 40. The concept of lebenstraum in Nazism related to:
(a) territory
(b) race
(c) folk
(d) leader
Ans (b)
Q. 41. Which one of the following is not an element of Democratic Socialism?
(a) Social Justice
(b) Revolutionary Change
(c) Mixed Economy
(d) Welfare State
Ans. (b)

Muftbooks.com

MCQs for political science optional aspirants


Q. 42. The author of History and Class Consciousness is:
(a) Althusser
(b) Gramsci
(c) Lenin
(d) Lukas
Ans. (d)
Q. 43. One of the weaknesses of Guild Socialism was that is:
(a) ignored the interests of consumers
(b) denied political power to industrial workers
(c) aimed at abolition of State by violent means
(d) unnecessarily justified territorial representation
Ans (a)
Q. 44. One of the aims of Fabian Socialism was to:
(a) emancipate industrial capital from private ownership
(b) train industrial workers for revolutionary action
(c) establish peacefully a classless and stateless society
(d) delegate political authority to autonomous guilds
Ans. (a)
Q. 45. Bernard Shaw, who was himself a leading light of this variety of socialism, said about them that they agreed to give up the
delightful ease of revolutionary heroics and to take to the hard work of practical reform on ordinary Parliamentary lines. What was that
variety of socialism?
(a) Utopian socialism
(b) Evolutionary socialism
(c) Guild socialism
(d) Fabian socialism
Ans. (d)
Q 46. Lenin reached the conclusion that the disintegration of capitalism would begin in the:
(a) Industrial heartland
(b) Tsarist Russia
(c) Colonial periphery
(d) USA
Ans. (c)
Q. 47. Which of the following pairs are correctly matched?
1. Human dignity Liberalism
2. Rule of law Fascism
3. Stateless Society Anarchism
4. Rigid state control Totalitarianism
(a) 1, 2, 3 and 4
(b) l, 3 and 4
(c) 2 and 3
(d) 1 and 4
Ans. (b)
Q. 48. Match List- I with List- II and select the correct answer:
List I List -II
(Socialism) (Propounder)
A. Utopian 1. Karl Marx
B. Fabian 2. G.D.H. Cole
C. Scientific 3. Sydney Webb
D. Guild 4. Saint Simon
Codes:
ABCD
(a) 3 4 1 2
(b) 4 3 2 1
(c) 3 4 2 1
(d) 4 3 1 2
Ans. (d)
Q. 49. Fabian economic theory is based in:
(a) Ricardian theory of rent

Muftbooks.com

MCQs for political science optional aspirants


(b) Labour theory of value
(c) Laissez-faire theory
(d) Theory and mixed economy
Ans (d)
Q. 50. Which one of the following is not a principal postulate of liberal movement?
(a) Individual freedom
(b) Powerful state
(c) Rule of law
(d) Political rights
Ans. (b)
......................................................-----------------------------................................................................
mcq Political science Solved Questions
MCQ Pol Science
Pol Science Objective questions solved
Q. 1. Which one of the following theories is known as Voices from periphery?
(a) Modernization
(b) Class-struggle
(c) Dependency
(d) Imperialism
Ans. (c)
Q. 2. New Dependency emphasizes on:
(a) surplus extraction as the major cause of underdevelopment
(b) monopoly of trade complemented by a monopoly of land, mines and manpower in developing countries
(c) domination of capital by hegemonic centers
(d) investment by multinational corporations
Ans. (d)
Q. 3. Given below are two statements, one labelled as Assertion (A) and the other labelled as Reason (R)
Assertion (A): Planning in advance for development Textiles and apparels are products in which developed countries have a
comparative advantage.
Reason (R): Textiles and apparels have been for long, a labour intensive industries.
In the context of the above two statements. Which one of the following is correct?
Codes:
(a) Both A and R are true and R is the correct explanation of A.
(b) Both A and R are true and R is not the correct explanation of A
(c) A is true but R is false.
(d) A is false but R is true.
Ans. (c)
Q. 4. Who used to concept of associated dependent development?
(a) Ernesto Laclau
(b) Alfred Stepar

Muftbooks.com

MCQs for political science optional aspirants


(c) Dos Snatos
(d) Fernando Cardosa
Ans. (d)
Q. 5. Environment problems and issues received special attention of the Government of India from the:
(a) Third Five-Year Plan
(b) Fourth Five-Year Plan
(c) Sixth Five-Year Plan
(d) Ninth Five-Year Plan
Ans. (c)
Q. 6. Consider the following events:
1. Nationalization of Banks
2. Pokhran-I
3. Statehood to Goa
4. Passing of the Hindu Code Bill
The correct chronological order of these events is:
(a) 4, 2, 3, 1
(b) 4, 1, 2, 3
(c) 3, 1, 2, 4
(d) 2, 1, 4, 3
Ans. (b)
Q. 7. Given below are two statements, one labelled as Assertion (A) and the other labelled as Reason (R):
Assertion (A): Regionalism in Indian politics reflects the cultural diversity of the country.
Reason (R): The Political leaders which up the ethic identities.
In the context of the above two statements, which one of the following is correct?
Codes:
(a) Both A and R are true and R is the correct explanation of A
(b) Both A and R are true but R is not the correct explanation of A
(c) A is true but R is false
(d) A is false but R is true
Ans. (b)
Q. 8. Given below are two statements, one labelled as Assertion (A) and the other labelled as Reason (R):
Assertion (A): The challenge that the Indian state is facing today is how to achieve an equivalence between the state and the civil
society within a democratic framework.
Reason (R): Criminalization of politics, political corruption and communal violence stand in the way of creating a value based political
culture.

Muftbooks.com

MCQs for political science optional aspirants


In the context of the above two statements, which one of the following is correct?
Codes:
(a) Both A and R are true and R is the correct explanation of A
(b) Both A and R are true but R is not the correct explanation of A
(c) A is true but R is false
(d) A is false but R is true
Ans (a)
Q. 9. Given below are two statements, one labelled as Assertion (A) and the other labelled as Reason (R):
Assertion (A): The parliamentary democratic system as adopted and operating in India is not capable of responding effectively to the
challenge of economic equalities.
Reason (R): The most explosive issues faced by Indias post-independence leadership have been the problems of caste oppressions
and socioeconomic inequalities.
In the context of the above two statements, which one of the following is correct?
Codes:
(a) Both A and R are true and R is the correct explanation of A
(b) Both A and R are true but R is not the correct explanation of A
(c) A is true but R is false
(d) A is false but R is true
Ans. (b)
Q. 10. Communalism in Indian context means:
(a) serving the community in a benefiting manner
(b) using communal identity for political gains
(c) a group of people bound by ethic feelings
(d) creating friendly relations with the other communities
Ans. (b)
Q. 11. In which year did India become a party to the International Convention on Biological Diversity?
(a) 1992
(b) 1993
(c) 1994
(d) 1995
Ans. (a)
Q. 12. As per the 2001 Census, the minimum standards of literacy are:
(a) a person aged 12 years and above who can read and understand any language
(b) a person aged 18 years and above who can read speak and write in any language
(c) a person aged 7 years and above who can both read and write with understanding in any language

Muftbooks.com

MCQs for political science optional aspirants


(d) a person aged 21 years and above, who can read, speak and write any language
Ans. (c)
Q. 13. Birsa Munda was associated with tribal struggle of:
(a) North East Region
(b) Jharkhand
(c) Nagpur Division
(d) Deccan
Ans. (b)
Q. 14. Given below are two statements, one labelled as Assertion (A) and the other labelled as Reason (R)
Assertion (A): Naxalite movement is an armed struggle of poor peasantry against the landlords.
Reason (R): Naxalite movement was born in the Naxalbari village of Darjeeling District.
In the context of the above two statements, which one of the following is correct?
Codes:
(a) Both A and R are true and R is the correct explanation of A
(b) Both A and R are true but R is not the correct explanation of A
(c) A is true but R is false
(d) A is false but R is true
Ans. (b)
Q. 15. Which one of the following is not a Peasant movement?
(a) Telengana Insurrection
(b) Tebhaga Movement
(c) Naxalvadi Movement
(d) Appiko Movement
Ans. (d)
Q. 16. Which one of the following is not an NGO campaigning for Human Rights?
(a) Amnesty International
(b) World Watch
(c) PUCL
(d) Oxfam
Ans. (d)
Q. 17. The well-known theorist of New Social Movement is:
(a) Habermas
(c) Foucault
(b) Karl Marx

Muftbooks.com

MCQs for political science optional aspirants


(d) Althusser
Ans. (c)
Q. 18. Which of the following pairs is not correctly matched?
(a) Sunderlal Bahuguna
(b) Sardar Patel

: Chipko Movement
: Warli Struggle

(c) Narayanaswamy Naidu : Tamilliga Vyvasaigal Sangam


(d) Ram Manohar Lohia

: Kagodu Satyagraha

Ans. (b)
Q. 19. Match List- I with List- II and select the correct answer:
List I (Individual)

List -II (Movement)

A. Sharad Joshi

1. Gandhian Movement

B. Dattopant Thengadi

2. Chipko Movement

C. Nirmala Deshpande

3. Farmers Movement

D. Sundar Lal Bahuguna

4. Workers Movement

Codes:
ABCD
(a) 3 4 1 2
(b) 3 1 4 2
(c) 2 4 1 3
(d) 2 1 4 3
Ans. (a)
Q. 20. Who among the following was the first President of AITUC?
(a) Joseph Baptista
(b) Lala Lajpat Rai
(c) B.P. Wadia
(d) P.C. Joshi
Ans. (b)
Q. 21. Who among the following is associated with Eco- Feminism?
(a) Suman Sahi
(b) Shabana Azmi
(c) Vandana Shiva
(d) Arundhati Roy
Ans. (c)
Q. 22. Who among the following was considered to be a forerunner of backward class movement?

Muftbooks.com

MCQs for political science optional aspirants


(a) Dr. B.R. Ambedkar
(b) Jyotirao Phule
(c) E.V. Ramsawamy Naicker
(d) Ram Manohar Lohia
Ans. (b)
Q. 23. Given below are two statements, one labelled as Assertion (A) and the other labelled as Reason (R):
Assertion (A): A social movement is a particular form of collective behaviour in which the motive to act springs from the attitude of
members and the challenges of the society.
Reason (R): Social Groups and movements embrace interest groups and political parties to achieve their goals.
In the context of the above two statements, which one of the following is correct?
Codes:
(a) Both A and-R are true and R is the correct explanation of A
(b) Both A and R are true but R is not the correct explanation of A
(c) A is true but R is false
(d) A is false but R is true
Ans. (b)
Q. 24. Generally Non-government Organization in India are registered under:
(a) Registration Act, 1860
(b) Registration Act, 1871
(c) Registration Act, 1960
(d) Registration Act, 1948
Ans. (a)
Q. 25. In which year was the Human Rights Protection Law passed in India?
(a) 1986
(b) 1987
(c) 1993
(d) 1995
Ans. (c)
Q. 26. Which one of the following statements is not true with regard to Non-Government Organizations?
(a) They have come to play increasingly constructive role in the development process
(b) They have to redefine their role in accordance with the changing times
(c) The government has to change the bureaucratic
(d) The government should affiliate Non-Government Organization to its respective departments in order to have better control over
their functioning
Ans (d)

Muftbooks.com

MCQs for political science optional aspirants


Q. 27. The scientific method does not emphasize on:
(a) empiricm
(b) values
(c) facts
(d) objectively
Ans. (b)
Q. 28. Jhum cultivation by cutting and burning trees is in practice by indigenous tribes in:
(a) Karnataka
(b) North-Eastern States
(c) Orissa
(d) Jammu and Kashmir
Ans. (b)
Q. 29. Which one of the following social movements is driven more by identity, concerns rather than the ideological concerns?
(a) Narmada movement
(b) Farmers movement Karnataka against the MNCs
(c) Sub caste reservation movements among the Scheduled Castes
(d) Anti-liquor shop campaign in Andhra Pradesh
Ans. (c)
Q. 30. Match List -I (Movements) with List- II (Issues) and select the correct answer using the codes given below the Lists:
List I

List-II

A. Panthers

1. Agrarian relations

B. Chipko

2. Environmental

C. Jharkhand

3. Exploitation discrimination

D. Tebhaga

4. Statehood

Codes:
ABCD
(a) 4 2 3 1
(b) 3 1 4 2
(c) 4 1 3 2
(d) 3 2 4 1
Ans. (d)
Q. 31. In the eyes of a radical feminist, gender equality can be achieved through:
(a) gradual reform
(b) getting more women elected in legislatures

Muftbooks.com

MCQs for political science optional aspirants


(c) challenging patriarchy
(d) class struggle
Ans (c)
Q. 32. Which one of the following is not a peasant movement?
(a) Kheda movement
(b) Tabligh movement
(c) Telengana movement
(d) Tebhaga movement
Ans. (b)
Q. 33. Which one of the following non-governmental organizations is exclusively involved in human rights advocacy?
(a) Oxfam
(b) Ford foundation
(c) Amnesty International
(d) Caritas
Ans. (c)
Q. 34. The most essential principle of liberalism is:
(a) Equality
(b) Social justice
(a) Freedom
(d) Democracy
Ans. (c)
Q. 35. Which one of the following doctrines rejects the role of the State as an instrument of social transformation?
(a) Guild socialism
(b) Syndicalism
(c) Fabian socialism
(d) Marxian socialism
Ans. (d)
Q. 36. The formula from each according to his ability to each according to his work for the system of distribution in the State was
given by:
(a) G.D.H. Cole
(b) Sorel
(c) H.G. Wells
(d) Lenin
Ans. (a)

Muftbooks.com

MCQs for political science optional aspirants


Q. 37. The Marxian theory of Surplus Value is largely derived from the theory of:
(a) Adam Smith
(b) Ricardo
(c) Malthus
(d) John Stuart Mill
Ans. (b)
Q. 38. Which one of the following is not true of Fabianism?
(a) It regards the transition from capitalism to socialism as a gradual and peaceful process through political agencies already at hand
(b) It sees in the middle class a group that can be utilized in developing the technique of administration on behalf of the new social
order
(c) It is an important step in rousing the social conscience of the community in favour of the socialist ideal
(d). It entirely retains the individual ownership of baise industries and system of production and distribution but seeks to control these
for social good
Ans. (d)
Q. 39. Which one of the following brands of socialism considers socialism as a refinement and logical conclusion of democracy:
(a) Fabianism
(c) Marxism
(b) Syndicalism
(d) Guild socialism
Ans. (a)
Q. 40. Who among the following thinkers has attempted to apply structuralism to the study of Marxist socialism?
(a) Gramsci
(b) Horkheimer
(c) Sartre
(d) Althuseer
Ans. (d)
Q. 41. Given below are two statements one labelled as Assertion (A) and the other labelled as Reason (R):
Assertion (A): Scientific socialism explains history in terms of class struggle.
Reason (R): Class struggle in history takes place as a result of contradictions between the proletariat and the peasantry.
In the context of the above two statements which one of the following is correct?
Codes:
(a) Both A and R are true and R is the correct explanation of A
(b) Both A and R are true but R is not a correct explanation of A
(c) A is true but R is false
(d) A is false but R is true

Muftbooks.com

MCQs for political science optional aspirants


Ans. (c)
Q. 42. Which of the following are true of socialism?
1. Socialism aims at the elimination of capitalism
2. Socialism stands for the subordination of the interests of the society to the higher interests of the individuals
3. Socialism stands for justice, fair play and liberty. It aims at adding to the sum total of social welfare
4. Socialism stands for the elimination of competition where there is much economic waste
(a) 1, 2 and 3
(b) 1, 3 and 4
(c) 1, 2 and 4
(d) 2, 3 and 4
Ans. (b)
Q. 43. Consider the following statements about the function of the State ascribed to the democratic socialists:
1. They favour collective control and a wide extension of public activities
2. They believe th individual freedom can be better secured
3. They believe in state ownership of the means of production for the social welfare of the people
4. They believe in strong and prosperous state as end in itself
Which of the above is/are correct? Choose the correct answer from the codes given below:
(a) 1, 2, 3 and 4
(b) 1, 2 and 3
(c) 1 and 2
(d) None
Ans. (b)
Q. 44. Who among the following stated the principle of fascism as a nationalist from of socialism?
(a) Martin Heidegger
(b) Georges Sorel
(c) Alfredo Rocco
(d) Rudolf Kjellen
Ans. (c)
Q. 45. The official philosopher of national socialism was:
(a) Rosenberg
(b) Heidegger
(c) Wagner
(d) Haushofer
Ans. (a)

Muftbooks.com

MCQs for political science optional aspirants


Q. 46. Which one of the following features distinguishes Nazism from Communism?
(a) Totalitarian state
(b) Lack of individual freedom
(c) Equality of all
(d) Secret police
Ans. (c)
Q. 47. Which one of the following statements enunciates a principle of totalitarianism?
(a) Over himself, over his own body a mind, the individual is sovereign
(b) He who does not work, neither shows be eat
(c) Force is the midwife of every society pregnant with a new one
(d) We live, move and have our being the State
Ans. (d)
Q. 48. Which of the following thinkers were exponent of Guild Socialism?
1. Robert Owen
2. Graham Wallas
3. Bertrand Russell
4. G.D.H. Cole
(a) l and 4
(b) l and 3
(c) 2 and 4
(d) 3 and 4
Ans. (d)
Q. 49. Lenins first important theoretical work relates to:
(a) factors of production
(b) organization of the party
(c) industrial economy
(d) peasant movements
Ans. (c)
Q. 50. A half-way house between States Socialism and Syndicalism
This description fits:
(a) Fabian Socialism
(b) Guild Socialism
(c) Marxian Socialism
(d) Platonic Communism

Muftbooks.com

MCQs for political science optional aspirants


Ans. (a)
......................................................................------------------------------..............................................
Political Science trivia questions and quiz
Political Philosophy Quiz Political Science
Quiz about Political Science. It is from previous papers of UPSC
political Science Exam Questions Quiz For UPSC WBPSC UPPSC PPSC HPSC TNPSC and other job exams
1. If an amendment of the Constitution of India seeks to make any change in certain items, the amendment requires to be ratified by
the Legislatures of not less than one-half of the States before the Bill making provision for such amendment is presented to the
President for assent.
Which of the following are those items?
A. Representation of States in Parliament.
B Any of the Lists in the Seventh Schedule.
C. Manner of election of the President of India.
Select the correct answer using the codes given below:
(a) A and B
(b) B and C
(c) A and C
(d) A, B and C
Ans. (d)
2. Which one of the following is not correct about Finance Commission?
(a) The members of the Commission are appointed by the President
(b) It is to act as a balancing wheel of Indian fiscal federalism
(c) The qualifications of members of the Commission are determined by the President
(d) It is constituted under the provisions of Article 280 of the Constitution of India
Ans. (c)
3. In which one of the following Five-Year Plans, was a State-wise break-up of the broad development targets, including targets for
growth rates and social development, which are consistent with the national targets, included for the first time?
(a) Seventh Five-Year Plan
(b) Eighth Five-Year Plan
(c) Ninth Five-Year Plan
(d) Tenth Five-Year Plan
Ans. (d)
4. For which, one of the following, has a provision not been explicitly stated in the Constitution of India?
(a) Inter-State Council.
(b) All India Services
(c) Contingency Fund of India
(d) National Development Council
Ans. (d)
5. Which of the following broad issues are included in the terms of reference for the 13th Finance Commission?
A. Sustainable development.
B. Tax-Gross Domestic Product ratio.
C. Goods and Service Tax.
Select the correct answer using the codes given below:
(a) A and B
(b) B and C
(c) A and C
(d) A, B and C
Ans. (c)
6. What is the proposed bade of population figures by the 13th Finance Commission?
(a) 1971
(b) 1981
(c) 1991
(d) 2001
Ans. (d)
7. Consider the following pairs:
A. Taxes levied by the Union : Stamp duties
but collected and appropriated by the State
B. Taxes levied and collected : Taxes on railway fares and freight

Muftbooks.com

MCQs for political science optional aspirants


by the Centre but assigned and freight to States
Which of the pairs given above is/are correctly matched?
(a) A only
(b) B only
(c) Both A and B
(d) Neither A or B
Ans. (c)
8. Which of the following issues are included in the terms of reference of the Commission of Centre-State.
Relations set up by the Government of India?
A. Inter-linking of rivers.
B. Devolution of powers and autonomy to Panchayat Raj Institutions.
C. Freeing Inter-State trade. Select the correct answer using the codes given below:
(a) A and B
(b) B and C
(c) A and C
(d) A, B and C
Ans. (a)
9. Which one of the following is the correct association?
(a) Indira Sawhney Case : Basic structure of the Constitution
(b) Minerva Mills Case : Invocation of Article 356
(c) Maneka Gandhi Case : Just, fair and reasonable procedure of law
(d) Pondyal vs. Union of India Case : Financial Emergency
Ans. (c)
10. Which one of the following is the correct statement?
Justice Ismail Commission was constituted to
(a) investigate Godhra violence
(b) study reservation for OBCs
(c) study the status of Navratna companies
(d) investigate police excesses in prisons
Ans. (d)
11. Which one of the following has been justified by the New Right?
(a) Free market society with welfare state
(b) Free -market society with minimal state
(c) Free market society with planning in detail
(d) Bureaucratic control and non-market economy
Ans. (a)
12. Chipko Movement, which drew strength from Sarvodaya Movement, in its different phases, was mainly which one of the following?
(a) Quintessential peasant movement and a celebrated environmental movement
(b) Successful commercial forestry in certain catchments
(c) Movement to prevent the loss of land
(d) Movement to retain underground water table
Ans. (a)
13. Match List -I with List- II and select the correct answer using the codes given below the Lists:
List -I List-II
(Electoral System) (Country)
A. Second Ballot System 1. Japan
B. Alternative Vote System (AVS) 2. Germany
C. Additional Member System (CAMS) 3. France
D. Limited Vote System 4. Australia
Code:
ABCD
(a) 2 1 3 4
(b) 3 1 2 4
(c) 3 4 2 1
(d) 4 2 3 1
Ans. (c)
14. Vishakha and others vs. State of Rajasthan Case pertained to which one of the following?
(a) Discrimination against women in the Police Service

Muftbooks.com

MCQs for political science optional aspirants


(b) Reservation in promotion for the Scheduled Castes and Scheduled Tribes in Government Service
(c) Reservation in promotion for the Other Backward Classes in Government Service
(d) Sexual harassment of women at work places
Ans. (d)
15. Alain Touraine is known for which one of the following?
(a) Literature on Social Movements
(b) Literature on international relations
(c) Multi-party systems in democracy
(d) Electoral reforms
Ans. (a)
16. Which one among the following is not a member of the G8 countries whose last Summit was held in Germany in June, 2007?
(a) Italy
(b) France
(c) Japan
(d) China
Ans. (d)
17. Which of the following are the most important components of the Treaty on European Union?
A. European Parliament.
B. The European Community
C. Common Foreign and Security Policy (C.E.S.P)
D. Justice and Home Affairs (J.H.A.)
Select the correct answer using the codes given below:
(a) A and C
(b) B and D
(c) A, B and D
(d) B, C and D
Ans. (c)
18. Who among the following has rejected the notion of a Third World, claiming that there is only one world connected by a complex
network of economic exchange relationships-a world economy?
(a) Immanuel Wallerstein
(b) Andre Gunder Frank
(c) Samir Amin
(d) Paul Baran
Ans. (a)
19. Which one of the following pairs is correctly matched?
Conference Issue
(a) Durban : South-South Dialogue
(b) Beijing : Gender Rights
(c) Uruguay Round : Disarmament
(d) Copenhagen : Global Warming
Ans. (b)
20. Which one of the following countries is not a one-party system today?
(a) Vietnam
(b) Peoples Republic of China
(c) Poland
(d) North Korea
Ans. (c)
21. Weberiarr bureaucracy is a less efficacious instrument of social change for developing countries. This criticism of Webers theory of
bureaucracy is attributed to whom among the following?
(a) Carl Friedrich
(b) Laurence J. Peter
(c) Paul Appleby
(d) Joseph La-Palombara
Ans. (d)
22. Match List I with List II and select the correct answer using the codes given below the Lists:
List-I List II
(Committee) (Objective)
A. Bardoli Committee 1.To introduce reforms in the electoral system

Muftbooks.com

MCQs for political science optional aspirants


B. Shourie Committee 2.To prepare a system of governance for the hill areas of
the North-East.
C. Goswami Committee 3. To study nexus between politicians and criminals
D. Chavan Committee 4. To examine feasibility of Bight to Information
5. To study measures to prevent defection of legislators
Code:
ABCD
(a) 5 4 1 2
(b) 2 1 3 5
(c) 5 1 3 2
(d) 2 4 1 5
Ans. (d)
23. Which one of the following recommended the abolition of the post of Governor of a State?
(a) The Rajamannar Committee
(b) The Sarkaria Commission
(c) The Governors Conference
(d) The First Administrative Reforms Commission
Ans. (a)
24. Match List-I with List-II and select the correct answer using the codes given below the Lists:
List I List -II
(Provision) (Contained in)
A. Provisions Concerning 1. Chapter I of Part V of the Constitution of India
Administration of Tribal Areas
B. Language of the Union 2. Part XIII of the Constitution of India
C. Trade and Commerce within India 3. Part XVII of the Constitution of India
D. The Executive of the Union 4. Sixth Schedule of the Constitution of India
Code:
ABCD
(a) 2 1 4 3
(b) 4 3 2 1
(c) 2 3 4 1
(d) 4 1 2 3
Ans. (b)
25. The temporary and transitional provisions mentioned in Part XXI of the Constitution of India relate to which State(s)?
1. Jammu and Kashmir
2. Nagaland
3. Assam
4. Andhra Pradesh
Select the correct answer using the codes given below:
(a) A only
(b) C and D
(c) A, B and C
(d) A, B, C and D
Ans. (d)
26. In the context of the Right to Information Act, 2005 which one of the following pairs is not correctly matched?
Body Competent Authority
(a) Supreme Court: Chief Justice of India
(b) Union Public-Service Commission: Chairman, Union Public Service Commission
(c) Lok Sabha: Speaker of Lok Sabha
(d) High Court of a State: Chief Justice of the High Court
Ans. (c)
27. Consider the following:
A. The budget allocated to each agency of a public authority
B. Particulars of recipients of concessions, permits or authorizations granted by a public authority. Information in respect of which of
the above is obligatory for the public authority to publish under the
Right to Information Act, 2005?
(a) A only
(b) B only
(c) Both A and B

Muftbooks.com

MCQs for political science optional aspirants


(d) Neither A nor B
Ans. (c)
28. Through which one of the following famous cases in American legal history did Justice John Marshall establish the doctrine of
Judicial Review?
(a) Dred Scott vs. Sanford
(b) Roe vs. Wade
(c) Marbury vs. Madison
(d) Baker vs. Can
Ans. (c)
29. Constitution (Seventy-third Amendment) Act that provided for the establishment of Panchayats also made which of the following
provisions?
A. Addition of a new Schedule (XI) to the Constitution listing 29 subjects which are to be handled by the Panchayats.
B. Addition of a new Schedule (XI) to the Constitution listing 18 subjects which are to be handled by the Panchayats.
C. Bar on the courts to question the validity of laws relating to delimitation of constituencies.
D. Number of seats in Panchayats, not to be less than one-third (excluding the number of seats reserved for women belonging to the
Scheduled Castes and Scheduled Tribes) be reserved for -women.
Select the correct answer using the codes given below:
(a) A, B and D
(b) A and C
(c) B and D
(d) A only
Ans. (b)
30. Consider the following statements:
A. Like the Judge of a High Court, the Advocate-General of a State is appointed by the President of India and holds office during the
pleasure of the President.
B. In the M.C. Mehta vs. Union of India Case (Kanpur Tanneries matter) the Supreme Court prioritized development over environmental
issues.
Which of the statements given above is/are correct?
(a) A only
(b) B only
(c) Both A and B
(d) Neither A nor B
Ans. (d)
31. Who among the following can attend the meetings of both Houses of Parliament while being not a member of either of the Houses?
(a) Solicitor-General of India (b) Vice-President of India
(c) Comptroller and Auditor-General of India (d) Attorney-General of India
Ans. (d)
32. Who Among the following appoints the Chairman of the Public Accounts Committee?
(a) Union Minister of Finance
(b) Leader of Opposition in the Lok Sabha
(c) Chairman of Rajya Sabha
(d) Speaker of Lok Sabha Arts.
Ans. (d)
33. Who among the following scrutinizes the Report of the Comptroller and Auditor-General of India?
(a) Public Accounts Committee
(b) Department of Expenditure, Union Ministry of Finance
(c) Controller General of Accounts
(d) Cabinet Secretariat
Ans. (a)
34. Match List -I with List-II and select the correct answer using the codes given below the Lists:
List List -II
(Subject) (Article of the Constitution of India)
A. Annual Financial Statement or budget 1.Article 110
B. Supplementary, additional or excess grants 2.Article 112
C. Appropriation Bill 3.Article 114
D. Definition of Money Bill 4.Article 115
Code:
ABCD

Muftbooks.com

MCQs for political science optional aspirants


(a) 2 3 4 1
(b) 1 4 3 2
(c) 2 4 3 1
(d) 1 3 4 2
Ans. (c)
35. If a member of either House of Parliament, is without the permission of the House absent from all meetings thereof for more than a
certain period, the House may declare his seat vacant. What is the duration of this period?
(a) 60 days
(b) 45 days
(c) 30 days
(d) 15 days
Ans. (a)
36. Consider the following statements in respect of the Supreme Court of India:
A. It has the power to deliver advisory opinion on any question of fact or law referred to it by the President.
B. Salaries of the Judges of the Supreme Court are subject to vote by Parliament.
C. Its jurisdiction is binding on all other courts within India.
Which of the statements given above are correct?
(a) A and B
(b) B and C
(c) A and C
(d) A ,B and C
Ans. (c)
37. In which one of the following cases has the Supreme Court observed that where public interest is undermined by an arbitrary and
perverse executive action, the High Court can issue a writ?
(a) Chaitanya vs. State of Karnataka
(b) High Court of Judicature at Bombay vs. S. K. Ranga Rao Patil
(c) State of West Bengal vs. Sampat
(d) Raunaq International Ltd. vs. I R. Construction Ltd.
Ans. (a)
38. Consider the following statements:
A. The Supreme Court of India can sit only in Delhi.
B. The Supreme Court of India can sit in places.other than Delhi which the Chief Justice of India may appoint with the approval of the
President.
C. The Parliament may, by law, make provisions for conferring supplemental powers upon the Supreme Court.
Which of the statements given above is/are correct?
(a) A and C
(b) B and C
(c) B only
(d) C only
Ans.( b)
39. Consider the following statements:
Under Article 200 of the Constitution of India, the Governor may:
A. withholds his assent to a Bill passed by the State legislature.
B. reserve the Bill passed by the State legislature for the consideration, of the President.
C. returns a Bill, other than a money Bill, for reconsideration of the legislature.
Which of the statements given above are correct?
(a) A and B
(b) B and C
(c) A and C
(d) A, B and C
Ans. (d)
40. Consider the following statements:
A. An amendment of the Constitution of India can be initiated by the introduction of a Bill only in the Lok Sabha.
B. The Bill for amendment of the Constitution of India has to be passed in each House by a majority of the total membership of that
House and by a majority of not less than two-thirds of the members of that House present and voting.
Which of the statements given above is/are correct?
(a) A only
(b) B only
(c) Both A and B

Muftbooks.com

MCQs for political science optional aspirants


(d) Neither A nor B
Ans.(b)
.............................................................---------------------------------,...........................................
upsc political science exam model sample paper
UPSC Question Papers
UPSC Model Question Papers
UPSC Sample Paper
UPSC Quiz : UPSC Prelims model papers : political science
Upsc Exam Model Papers political science
1. Consider the following with respect to the Union of India:
A. Formation of the State of Nagaland.
B. States Reorganization Act.
C. Formation of the State of Haryana.
D. Formation of the State of Gujarat.
Which one of the following is the correct chronological order of the above?
(a) D, B, C, A
(b) B, D, C, A
(c) D, B, A, C
(d) B, D, A, C
Ans. (d)
2. Consider the following pairs in respect of the General Elections 2004 to the 14th Lok Sabha:
A. Constituencies for : 80% (Approximately), of the total 543 seats
General Candidates:
B. Constituencies reserved for : 13.33% (Approximately), of the total 543 seats
Scheduled Castes Candidates
C. Constituencies reserved for : 6.66% (Approximately), of the total 543 seats
Scheduled Tribe Candidates
Which of the above pairs is/are not correctly matched?
(a) A only
(b) B and C
(c) A and C
(d) A, B and C
Ans. (d)
3. The judgment in the S.R. Bommai vs. Union of India case pertained broadly to which
Article of the Constitution of India?
(a) Article 29
(b) Article 32
(c) Article 353
(d) Article 356
Ans. (d)
4. Consider the following statements:
A. The National Commission for Scheduled Tribes was constituted under the
Constitution (Eighty -ninth Amendment) Act.
B. The first National Commission for Scheduled Tribes was constituted on October 2,
2000.
Which of the statements given above is/are correct?
(a) A only
(b) B only
(c) Both A and B
(d) Neither A nor B
Ans. (d)
5. What is the minimum population below which Panchayats at the intermediate level
are not constituted as per provision of the Constitution (Seventy-third Amendment)
Act?
(a) 20 lakh
(b) 25 lakh
(c) 30 lakh
(d) 35 lakh
Ans. (a)

Muftbooks.com

MCQs for political science optional aspirants


6. Consider the following statements:
A. The Chairperson of a Panchayat at district level is elected in such manner as the
Legislature of the State may, by law, provide.
B. Legislature of a State may, by law, provide for representation of members of the
Legislative Assembly of the State representing constituencies which comprise wholly
or partly a Panchayat area at a level other than the village level, in such Panchayat.
C. The Lok Sabha may, by law, provide for the representation of its members
representing constituencies which comprise wholly or partly a Panchayat area at a
level other than the village level, in such Panchayat.
Which of the statements given above is/are correct?
(a) A and B
(b) B only
(c) C only
(d) B and C
Ans. (b)
7. Match List-I with List II and select the correct answer using the codes given below the Lists:
List- I List-II
(Subject in the (Similarity with which
Constitution of India) other Constitution)
A. Provisions relating to Emergencies 1. American Constitution
B. Directive Principles of State Policy 2. Constitution of the German Reich
C. Fundamental Rights 3. United Kingdom
D. Parliamentary System Government of 4. Constitution of Eire
Code:
ABCD
(a) 2 4 3 1
(b) 42 1 3
(c) 2 4 1 3
(d) 4 2 3 1
Ans. (c)
8. Who among the following was not a member of the National Commission to Review the Working of the Constitution constituted by
the Government of India in the year 2000?
(a) Soli J. Sorabjee
(b).C. R. Irani
(d) P A. Sangma
(d) Sonmath Chatterjee
Ans. (d)
9. Which of the following were the main areas of concern for the National Commission to Review the Working of the Constitution
constituted by the Government of India in the year 2000?
A. Electoral Reforms.
B. Union-State Relations.
C. Enlargement of Fundamental Rights.
D. Administrative System and Standards in Public Life.
Select the correct answer using the codes given below:
(a) A and B
(b) B and C
(c) A and C
(d) A, B, C and D
Ans. (d)
The following items consist of two statements, One labeled the Assertion A and the other labeled the Reason R. You are to examine
these two statements carefully and decide whether the Assertion A and the Reason R are individually true and if so, whether the reason
is a correct explanation of the Assertion. Select your answers to these items using the codes given below and mark your answer sheet
accordingly:
Codes:
(a) Both A and Rare true and R is the correct explanation of A.
(b) Roth A and R are true but R is NOT a correct explanation of A
(c) A is true but R is false
(d) A is false but R is true
10. Assertion (A): The Commission on Centre-State Relations has been constituted by the Government of India by amending the
Constitution of India, and thus a constitutional status has been bestowed on it.
Reason (R): Issues like role of Governors, economic and social planning which are a part of the terms of reference of the Commission

Muftbooks.com

MCQs for political science optional aspirants


are very important issues in the governance of India.
Ans. (d)
11. Assertion (A) : Not contented with merely laying down the fundamental principles of governance, the authors of the Indian
Constitution followed the Government of India Act, 1919 in providing matters of administrative details.
Reason (R): The authors of the Indian Constitution had the apprehension that in the prevailing conditions of the country at that time,
the Constitution might be subverted unless the form of administration was also included.
Ans. (d)
12. Consider the following commissions:
A. Kaka Kalelkar Commission on Backward Classes.
B. Sarkaria Commission on Centre-State Relations.
C. Shah Commission on emergency excesses.
Which one of the following is the correct chronological order of the above?
(a) B, C, A
(b) A, B, C
(c) B, A, C
(d) A, C, B
Ans. (d)
13. Consider the following statements:
A. The first National Commission for Scheduled Castes and Scheduled was constituted in
the year 1992.
B. The First National Commission for Scheduled Castes and Scheduled Tribes was
constituted under the Constitution (Sixty-fifth Amendment) Act.
C. H. Hanumanthappa was the Chairman of the first National Commission for Scheduled
castes and Scheduled Tribes.
Which of the statements given above are correct?
(a) A and B
(b) B and C
(c) A and C
(d) A, B and C
Ans. (a)
14. Which of the following terms was not included in a Union of Trinity by Dr. B.R.
Ambedkar in his concluding speech in the Constituent Assembly?
(a) Liberty
(b) Flexibility
(c) Equality
(d) Fraternity
Ans. (b)
15. In which one of the following countries, do units of the federation not have equal representation in the Upper House of the Central
Legislature?
(a) India
(b) Switzerland
(c) Australia
(d) United States of America
Ans. (a)
16. In the United States of America, who among the following uses/use filibuster?
(a) The Congress to pressurize the President
(b) The President to prevent the introduction of a bill in the Congress
(c) Members of the Senate to obstruct the passage of a bill
(d) Employers to break up an employees strike
Ans. (c)
17. Who among the following is vested with power to recall or remove officials such as
the President and the Vice-President of peoples Republic of China?
(a) The Standing Committee of the National Peoples Congress
(b) Chairman of the Central Military Commission
(c) The National Peoples Congress
(d) The President of the Supreme Peoples Court
Ans. (c)

Muftbooks.com

MCQs for political science optional aspirants


18. The President of South Africa is elected by whom among the following?
(a) National Assembly
(b)National Assembly and National Council of Provinces
(c) President of Constitutional Court
(d) Citizens of the country in a direct election
Ans. (a)
19. Regional devolution in Britain falls short of federalism because of the supremacy of
which one of the following?
(a) British Constitution
(b) British Parliament
(c) British Supreme Court
(d) European Commission
Ans.(b)
20. According to the Conservatives, the British Parliamentary institutions cannot be
transplanted into India for which of the following reasons?
(a) These institutions are the cultural expressions of British society
(b) India is a homogeneous society conducive to a Presidential System of Government
(c) A bi-party system is absolutely necessary for the effective functioning of British
Parliamentary System
(d) The role of British Monarch is critical for the smooth functioning of its parliamentary
institutions
Ans. (a)
21. Who among the following has/have identified rule-making, rule application and rule
adjudication as three authoritative governmental functions?
(a) Almond and Coleman
(b) Robert Dahl
(c) Harold J. Laswell
(d) David Easton
Ans. (a)
22. Which one of the following pairs is correctly matched?
(a) Structural functional approach : Bosanquet
(b) Behavioral approach : Charles Merriam
(c) Conflict approach : Seymour Lipset
(d) Marxist approach : Jacques Derrida
Ans. (b)
23. Partial responsible governments in the provinces had been established under which one of the following Acts?
(a) The Government of India Act, 1919
(b) The Government of India Act, 1935
(c) Indian Councils Act, 1909
(d) Indian Councils Act, 1892
Ans. (a)
24. According to the Cabinet Mission Plan, which one of the following was the method of electing the Constituent Assembly?
(a) Universal Adult Franchise
(b) Partly elected by the Provincial Assemblies and partly nominated by the Governor-General
(c) Nominated by the Princely States
(d) Partly elected by the Provincial Assemblies and partly nominated by the Princes
Ans.(d)
25. What were the salient features of the Government of India Act, 1935?
A. Provincial Autonomy.
B. Dyarchy at the Centre.
C. Abolition of Dyarchy in the States.
D. Retention of Excluded Areas.
Select the correct answer using the codes given below:
(a) B and C
(b) A, B and D
(c) A, C and D
(d) A, B, C and D
Ans. (d)

Muftbooks.com

MCQs for political science optional aspirants


26. Directive Principles of State Policy direct the State for which of the following?
A. To secure a social order for the promotion of welfare of the people.
B. Separation of judiciary from executive.
C. To improve public health.
Select the correct answer using the codes given below:
(a) A and B
(b) B and C
(c) A and C
(d) A, B and C
Ans. (d)
27. Which of the following is not included in Article 19 of the Constitution of India, pertaining to the Right to freedom?
(a) Right to reside and settle any part of the territory of India
(b) Right of minorities to establish and administer educational institutions
(c) Right to form associations of unions
(d) Right to assemble peaceably and without arms
Ans. (b)
28. Which one of the following writs is issued by the courts for release of a person unlawfully detained?
(a) Qua Warranto
(b) Prohibition
(c) Habeas Ct
(d) Certiorari
Ans. (c)
29. Which one of following is not correctly matched?
Subject Article of the Constitution of India
(a) Abolition of Untouchability Article 17
(b) Reservation of appointments Article 16(4)
or posts in favour of any backward class of citizens
(c) Prohibition of discrimination Article 18
on grounds of religion, race, caste, sex or place of birth
(d) Laws inconsistent with or in Article 13
derogation of the fundamental rights
Ans. (c)
30. What is the main object of protective discrimination?
(a) To give special advantage to a particular group
(b) To raise a disadvantaged group to a level where they can compete with other sections of society on equal footing
(c) To give financial assistance to the poor
(d) To promote class interest
Axis. (b)
31. Which Article of the Constitution of India directs the State to make effective provision for securing the Right to work?
(a) Article 16
(b) Article 38
(c) Article 41
(d) Article 43
Ans. (c)
32. Before giving any decision on questions as to disqualification of a member of either House of Parliament, the President of India shall
obtain the opinion of which one of the following?
(a) Supreme Court
(b) Election Commission
(c) Attorney-General of India
(d) Solicitor-General of India
Ans. (b)
33. Which one of the following is the correct statement?
Proclamation of Emergency in a State under Article
356 can continue beyond one year if:
(a) The High Court of the State certifies that the situation in the State in very serious
(b) The Governor of the State certifies that the law and order situation is not under control
(c) The Election Commission certifies that it is difficult to hold elections to the Assembly

Muftbooks.com

MCQs for political science optional aspirants


(d)The President is satisfied about the grave situation in the State through independent investigation
Ans. (c)
34. Consider the following statements:
A. No Proclamation of Emergency can be made by the President of India unless the Union Ministers of Cabinet rank, headed by the
Prime Minister, recommended to him, in writing, that such, a Proclamation should be issued.
B. The President of India must issue a Proclamation of Revocation of Emergency ar time that the Lok Sabha passes a resolution
.disapproving the proclamation or its continuance.
Which of the statements given above is / are correct ?
(a) A only
(b) B only
(c) Both A and B
(d) Neither A nor B
Ans.(c)
35. Which of the following statements dose not reflect the jurisdiction of the Committee on Estimates?
(a) It suggests alternative policies in order to bring efficiency and economy in administration
(b) It examines whether the money is well laid out within the limits of the policy implied in the estimates
(c) It suggests the form in which estimates are to be presented to Parliament
(d) It helps in checking Governmental extravagance in making demands as its report is debated in the Houses
Arts. (d)
36. Consider the following statements:
A. The Parliamentary Committee on Empowerment of Women was constituted for the first time after formation of the Fourteenth Lok
Sabha.
B. Margaret Alva was appointed as the Chairperson of the Parliamentary Committee on Empowerment of Women after the formation of
the Fourteenth Lok Sabha.
C. The Parliamentary Committee on Empowerment of Women consists of 20 members of the Lok Sabha and 10 members of the Rajya
Sabha.
Which of the statements given above is correct?
(a) A only
(b) B only
(c) C Only
(d) None of these
Ans. (c)
37. Which one of the following is the correct sequence through which an Ordinary Bill passes through in the Parliament ?
(a) Authentication, Discussion of Principles, Clause-by-Clause consideration, Post-introduction motion, Assert
(b) Post-introduction motion, Discussion of Principles, Clause-by-Clause consideration, Authentication, Assent
(c) Discussion of Principles, Authentication, Post-introduction motion, Clause-by-Clause consideration, Assent
(d) Discussion of Principles, Post-introduction motion, Clause-by-Clause consideration, Authentication, Assent
Ans. (b)
38. Consider the following statements:
A. The Parliament is empowered to alter the boundaries of any existing State of India.
B. A Bill pertaining to the alteration of the boundaries of any existing State of India can be introduced only in the Rajya Sabha and only
on the recommendation of the President of India.
Which of the statements given above is /are correct?
(a) A only
(b) B only
(c) Both A and B
(d) Neither A and B
Ans. (a)
39. Consider the following statements:
A. A Bill pending in the Rajya Sabha which has not been passed by the Lok Sabha shill lapse on a dissolution of the Lok Sabha.
B. A Bill pending in Parliament shall lapse by reason of the prorogation of the Houses.
Which of the statements given above is/are correct?
(a) A only
(b) B only
(c) Both A and B
(d) Neither A or B
Ans. (d)

Muftbooks.com

MCQs for political science optional aspirants


40. Assertion (A) : Recommendations of the Rajya Sabha, after a Money Bills is passed by the Lok Sabha and transmitted to the Rajya
Sabha have to be accepted by the Lok Sabha within 14 days from receipt of the recommendations and then get them incorporated in
the Bill.
Reason (R) Money Bill cannot be introduced in the Rajya Sabha.
Ans. (B)
...............................................-----------------------------------.......................................................
upsc political science solved questions and answers
Preliminary examination 2009 Solved questions with answers Pol Science
UPSC Political Science solved Questions and Answers
UPSC Question Papers Answers (Solved) political science model paper
1. Who among the following made the statement
Sovereignty is the supreme power of State, over citizens and subjects unhindered by law?
(a) J. Bodin
(b) H. Gardner
(c) Harold J. Laski
(d) Ayatollah Rohallah Khomeini
Ans. (a)
2. Which one of the following is the correct statement?
In the Pluralist Theory:
(a) the State is one among several associations
(b) the State is the sovereign association
(c) the State is subordinate to other associations
(d) the State includes the whole society
Ans. (a)
3. The idea of equality is not manifest in the thoughts of which one among the following?
(a) Eric Vogelin
(b) Julius K. Nyerere
(c) R.H. Tawney
(d) George Bernard Shaw
Ans. (a)
4. Social democracy articulates its concern for the weak and vulnerable on the basis of which of the following?
(a) Uniform civil code, State ownership of industry and public health:
(b) Welfarism, redistribution and social justice
(c) Trade unions, markets and more political parties
(d) Unity, heroism and anti-capitalism
Ans. (b)
5. Who among the following has expressed that an important view of representative democracy is that it is a mode of selecting leaders
from a given set of elites who compete for peoples vote?
(a) Dr. B. R. Ambedkar
(b) J. Schumpeter
(c) Robert Dah l
(d) Norberto Bobbio
Ans (b)
6. Which one of the following is the correct statement?
Marx and Engels suggest that under Primitive Communism there was no State because
(a) there were no classes and class conflict
(b) society was regulated by customs and tradition
(c) ruling class did not want a neutral agency such as State
(d) productive forces were equally shared
Ans. (a)
7. The problem of a just society is not the single problem of economic equality but the much more difficult problem of achieving
simultaneously in one society both liberty equality.
Which system does this passage justify.
(a) Capitalism
(b) Communism
(c) Fascism
(d) Democratic Socialism
Ans. (d)

Muftbooks.com

MCQs for political science optional aspirants


8. According to Marx, which one of the following is the principle of distribution in the Socialist State?
(a) From each according to his ability to each according to his needs
(b) From each according to his ability to each according to his work
(c) That property should be equally distributed among all
(d) That merit alone should be the basis of distribution
Ans. (b)
9. Match List- I with List-II and select the correct answer using the codes given below the Lists:
List- I List -II
(Marxs Writing) (Central Idea)
A. 18th Brumaire of Louis Bonaparte 1. Method
B. Paris Commune 2. Critique of Utopian Socialism
C. Grundrisse 3. Analysis of Politics
P. Poverty of Philosophy 4. Dictatorship of the Proletariat
Code:
ABCD
(a) 3 4 1 2
(b) 1 2 3 4
(c) 3 2 1 4
(d) 1 4 3 2
Ans. (a)
10. In Marxist thought, the Concept of Hegemony is closely associated with whom among the following?
(a) F. Engels
(b) Rosa Luxemburg
(c) A. Gramsci
(d) Mao-Tse-Tung
Ans. (c)
11. Which one of the following principles is rejected by Fascism?
(a) Racial/national superiority
(b) Obedience and personal loyalty to the leader
(c) Organic solidarity
(d) Rights and liberties
Ans.(d)
12. The subordination of the individual to the State is an essential feature of which one of the following?
(a) Socialist doctrine
(b) Feudal doctrine
(c) Conservative doctrine
(d) Fascist doctrine
Ans. (d)
13. Which among the following ideologies views government and all forms of institutionalized authority as a constraint on individual
liberty?
(a) Liberalism
(b) An Archaism
(c) Individualism
(d) Guild Socialism
Ans.(b)
14. Which one of the following is the correct statement? Totalitarian State is the one which
(a) commands unquestioning obedience to the authority of the ruler
(b) guarantees conditions In which the individual can realize his true self
(c) claims to regulate every sphere of individuals life and activities
(d) as a natural organism possesses a will of its own to promote a good life
Ans. (c)
15. What amongst the following is the main thrust of Neo-liberalism?
(a) To place market efficiencies as foundation for political freedom and to argue for a limited role for the State
(b) Commitment to an autonomous attitude to State, society and individual
(c) To accord priority to the will of the majority
(d)To extend power of a State beyond its borders
Ans.(a)

Muftbooks.com

MCQs for political science optional aspirants


16. Consider the following statements:
A. World-system analysis is a critique of both modernization theory and dependency theory.
B. In the world system approach economic power is closely linked with political and strategic clout.
C. States, which try to remain outside the orbit of the world economy find that they are unable to do so being dependent upon it for
trade, investment, technical assistance and aid.
Which of the statements given above is/are correct?
(a) A only
(b) A, B and C
(c) B and C
(d) A and C
Ans. (b)
17. Which of the following trends typify globalization?
A. The growing integration of national economies.
B. Political instability
C. Increased flow of information.
D. A growing geographical interdependence.
Select the correct answer using the codes given below:
(a) A and B
(b) B and D
(c) A,B and D
(d) A, C and D
Ans.(d)
18. Which one of the following cannot be considered a theorist of democracy?
(a) Alexis de Tocqueville
(b) J.J. Rousseau
(c) J.S. Mill
(d) E.W. Nietzsche
Ans. (d)
19. Consider the following statements:
A. Robert Keohana and Joseph Nye have discussed the changes in global environment resulting from the information age in their theory
of complex interdependence in Intentional relations.
B. The theory of complex interdependence is an opposing theory to neorealism in International relations.
Which of the statements given above is/are correct?
(a) A only
(b) B only
(c) Both A and B
(d) Neither A nor B
Ans. (c)
20. Which one of the following statements is not applicable to Morgehthaus theory?
(a)The master key is the concept of interest defined in terms of power
(b) Power is mans control over the minds and actions of men
(c) Political realism takes a fixed meaning of interest
(d) Politics is governed by objective laws having roots in human nature
Ans.(c)
21. Which one of the following principles is not endorsed by the Realist theory in International relations?
(a) Principal actors in the International system are sovereign states
(b) States are national actors engaged in pursuing their national interests
(c) Overriding goal of each state is its security and survival
(d) State survival is guaranteed best by adopting strategies of co-operation
Ans. (d)
22. Consider the following statements and identify the one which is not a correct representation of Morgehthaus ideas?
(a) Morgenthau presents a realist theory of international relations
(b) Morgenthau maintains that national interest and morality must determine State action
(c) Morgenthau argues that national interest transcends all principles of morality
(d) Morgenthau claims that national interest must guide the foreign policy of a State
Ans. (b)
23. Who among the following have emphasized the environmental factor in decision-making approach?
(a) Graham Allison and Ole Holsti

Muftbooks.com

MCQs for political science optional aspirants


(b) Talcott Parsons and Edward Shils
(c) Harold and Margaret Sprout
(d) Herbert Simon and J.W Burton
Ans. (c)
24. Consider the following statements about citizenship.
A. Citizenship confers on a person the membership in a political community:
B. In Democratic Societies, a citizen is one who enjoys the right to participate in the political process.
C. Exercising the right to vote is an essential condition for being a citizen in a Democratic Polity
D. Most democracies today grant citizenship only to those persons who were born within their territory
Which of the statements given above is/are correct?
(a) A only
(b) B and C
(c) A and B
(d) A, B, C and D
Ans. (c)
25. Over himself; over his mind and body, the individual is sovereign. Who among the following made this statement?
(a) H.J. Laski
(b) Immanuel Kant
(c) J.S. Mill
(d) G.W.F. Hegel
Ans. (c)
26. In Rawls theory, rights are based on which of the following principles?
(a) Deontologism
(b) Hedonism
(c) Pragmatism
(d) Idealism
Ans. (a)
27. Who among the following is not a proponent of the theory of natural rights?
(a) Thomas Hobbes
(b) John Locke
(c) Thomas Jefferson
(d) Jeremy Bentham
Ans. (d)
28. Who among the following argued that justice is the first virtue of social institutions?
(a) G.W.F. Hegel
(b) Karl Marx
(c) Charles Taylor
(d) John Rawls
Ans. (d)
29. What is negative liberty?
(a) Absence of constraints of law
(b) Absence of external hindrances
(c) Freedom to pursue ones own good
(d) Freedom to choose
Ans. (a)
30. Which one of the following pairs is not correctly matched?
(a) J. Bentham : Legal Rights
(b) J. Locke : Natural Rights
(c) I. Kant : Moral Rights
(d) G.W.F. Hegel : Cultural Rights
Ans. (d)
31. Who among the following viewed the State as an embodiment of the collective consciousness of the people cultures?
(a) WE Hegel
(b) John Locke
(c) Immanuel Kant
(d) J. Bodin
Ans. (a)

Muftbooks.com

MCQs for political science optional aspirants


32. Match List-I with List-II and select the correct answer using the codes given below the Lists:
List-I List -II
(Perspective) (Thinker)
A. The Neo-liberal Perspective 1. John Rawls
B. The Liberal Perspective 2. Edward Said
C. The Marxist Perspective 3. Robert Nozick
D. The Third World Perspective 4. Rosa Luxemburg
Code:
ABCD
(a) 4 2 3 1
(b) 3 1 4 2
(c) 4 1 3 2
(d) 3 2 4 1
Ans. (b)
33. Match List- I with List-II and select the correct answer using the codes given below the Lists:
List I List -II
(Theorist) (Approach)
A. G.H. Sabine 1. Historical Approach
B. G.E.G. Catlin 2. Sociological Approach
C. Leo Strauss 3. Philosophical Approach
D. Graham Wallas 4. Psychological Approach
Code:
ABCD
(a) 1 2 3 4
(b) 3 4 1 2
(c) 1 4 3 2
(d) 3 2 1 4
Ans. (a)
34. Match List I with List II and select the correct answer using the codes given below the Lists:
List I List -II
A. The greatest happiness of the 1.H. Green
greatest number
B. Removal of hindrances to 2. J. J. Rousseau
good life is the function of
the State
C. Man is born free but 3. J. Bentham
everywhere he is in chains
4. J. S. Mill
Code:
ABC
(a) 3 1 2
(b) 2 4 1
(c) 3 2 1
(d) 2 1 4
Ans. (a)
35. Which one of the following pairs of author and book is correctly matched?
(a) John Locke : Leviathan
(b) J.S. Mill : Representative Government
(c) J.J. Rousseau : Two Treatises on Government
(d) Thomas Hobbes : Social Contract
Ans. (b)
36. Which political thinker advocated plural voting to the higher educated citizens?
(a) J. Bentham
(b) J.J. Rousseau
(c) J.S. Mill
(d) T.H. Green
Ans. (c)
37. Who among the following held the view that the State comes into existence for the sake of life and it continues for the sake of
good life?
(a) J. Bentham

Muftbooks.com

MCQs for political science optional aspirants


(b) T.H. Green
(c) Aristotle
(d) Plato
Ans. (c)
38. Match List-I with List-II and select the correct answer using the codes given below the Lists:
List-I List-II
(Protagonist) (Concept)
A. Dworkin 1. Liberalism
B. Sandel 2. Comunitarianism
C. Maria Mies 3. Leo-feminism
D. Foucault 4. Post-structuralism
Code:
ABCD
(a) 3 4 1 2
(b) 1 2 3 4
(c) 3 2 1 4
(d) 1 4 3 2
Ans. (b)
39 Which one of the following pairs is correctly matched?
(a) Rawls: Impersonally best outcomes
(b) Nozick: Just initial acquisitions
(c) Hayek: Social redistribution
(d) Bentham: Equal opportunities and resources
Ans. (a)
40. Match List-I with List-II and select the correct answer using the codes given below the Lists:
List-I List -II
(Thinker) (Concept)
A. Mao Tse-Tung 1. One-dimensional Man
B. Antonio Gramsci 2. New Democracy
C. Jurgen Habermas 3. Autonomy of Super Structures
D. Herbert Marcuse 4. Communicative Rationality
Code:
ABCD
(a) 2 1 4 3
(b) 4 3 2 1
(c) 2 3 4 1
(d) 4 1 2 3
Ans. (c)

Muftbooks.com

You might also like